• Shuffle
    Toggle On
    Toggle Off
  • Alphabetize
    Toggle On
    Toggle Off
  • Front First
    Toggle On
    Toggle Off
  • Both Sides
    Toggle On
    Toggle Off
  • Read
    Toggle On
    Toggle Off
Reading...
Front

Card Range To Study

through

image

Play button

image

Play button

image

Progress

1/203

Click to flip

Use LEFT and RIGHT arrow keys to navigate between flashcards;

Use UP and DOWN arrow keys to flip the card;

H to show hint;

A reads text to speech;

203 Cards in this Set

  • Front
  • Back
(Q96 Aug 2008) [?Aug12] While of the following statements regarding patients with ankylosing spondylitis are FALSE

A amyloid renal infiltration is rarely seen

B cardiac complications occur in <10% of cases

C normovolaemia anaemia occurs in over 85% of cases

D sacroileitis is an early sign of presentation

E uveitis is the most common extra articular manifestation
C normovolaemia anaemia occurs in over 85% of cases
-
From wiki:
A TRUE Amyloidosis is a very rare complication of ankylosing spondylitis in patients with severe, active, and long-standing disease. These patients generally have active spondylitis, active peripheral joint involvement, and an elevated erythrocyte sedimentation rate (ESR) and C-reactive protein level. This may result in renal dysfunction with proteinuria and renal insufficiency or failure.[5]

B TRUE Cardiovascular involvement of clinical significance occurs in fewer than 10% of patients, typically those with severe long-standing disease. However, subclinical disease can be detected in many patients and may occur as an isolated clinical entity in association with HLA-B27[6]

C FALSE Approximately 15% of patients may present with a normochromic normocytic anemia of chronic disease.[7]

D TRUE

E TRUE Uveitis is the most common extra-articular manifestation, occurring in 20-30% of patients with ankylosing spondylitis. Of all patients with acute anterior uveitis, 30-50% have or will develop ankylosing spondylitis. The incidence is much higher in individuals who are HLA-B27–positive (84-90%).[8] --SG 10:37, 23 Oct 2008 (EDT)
-----------
++ [Aug12] New: ?15% full thickness burns 6 hrs ago in a child weighing 20kg. How much fluid to give in first hour?

A. ?600 mls
Ultimately depends on how question is worded...
-
from 'Burns in Children' CEACCP 2007 article, Table 1:
Parkland formula
For the first 24 h after the burn, give 4ml/kg per % BSA burn Hartmann's solution, half of this volume in the first 8 hour post-burn, the other half in the next 16 hours.
Maintenance - 4-2-1
Urine output - monitor and aim for 1mL/kg/hr

For this patient:
First 24 hours = 4x15x20 = 1200 mLs
Give 600 mLs in first 8 hours
PLUS maintenance of 60ml/hr - so in first 8 hours will need 480mLs of maintenance

So if this patient has had NO fluid resus - needs 600+(60x8) = 1080mLs in first 8 hours.

Can divide this volume over two hours - 540ml/hr??
-----------
++ [Aug12] New: Incarcerated inguinal hernia in a child with a mild URTI. Most appropriate course of action?

A. Postpone for 2 weeks
B. Continue without ETT
C. Continue with careful monitoring
C. ?
--
Incarcerated hernia is emergency surgery therefore cannot be postponed. URTI increases risk of adverse respiratory events.
-----------
++[Aug12] New: Patient with subdural hematoma and PPM for ?AV ablation. PPM technician >1 hour away. Surgeon wishes to proceed immediately. Do you?

A. Postpone and await a cardiologist review
B. Postpone and await arrival of PPM technician
C. Postpone and insert a transvenous temporary PM
D. Proceed after institution of transcutaneous pacing.
E. Proceed with a magnet handy.
B. Postpone and await arrival of PPM technician

IF URGENT

D. Proceed after institution of transcutaneous pacing
-
see ANZCA webinar on pacemakers (part 3); You cannot assume that a magnet will automatically switch it to asynchronous mode - the magnet function depends on what it was programmed to do. You would need to interrogate it to find out. The majority of the time it will switch it to asynchronous mode, HOWEVER some PPM have been programmed to ignore the magnet!


-----------
++[Mar12][Aug12] NEW:Middle-aged male with severe mitral stenosis {MS) having general anaesthesia for repair of fractured ulna / radius. 10 minutes into the case you notice a tachyarrythmia with his HR 130 and BP 70. He is normally in sinus. What do you do?

A. Adenosine
B. Amiodarone
C. Shock
D. :Volume
E. Metaraminol
c. Shock
--
Unsure if it is SVT or VT so shock!

a. Adensine - but if VT useless
b. Amiodarone - takes too lung
d. Volume - ?
e. metaraminol - ?if you think tachyarrhytmia is due to hypotension?

Depends on how this question is interpreted overall.
-----------
++[Mar12][Aug12] You are asked by an Obstetrician to help relax a uterus in labour and deliver for manual removal of placenta. What is a safe and effective dose of IV GTN to be delivered?

a. 5 mcg
b. 50 mcg
c. 250 mcg (or 200mcg in Aug12)
d. 400 mcg
e. 500 mcg
b. ?50mcg (safe) - tend to give in 50mcg boluses.

c. 200mcg? quoted dose sometimes is 100-200mcg
-----------
++[Oct08][Oct09][Sep11][Aug12]

Preoperative assessment shows a Mallampati (ML) score of III and thyromental distance (TMD) of < 6cm. A grade 3 to 4 on Cormark and Lehane is predicted. Compared to the ML score, the TMD is

A. less sensitive, less specific

B. less sensitive, more specific

C. more sensitive, less specific

D. more sensitive, more specific

E. equal sensitivity an specificity
B less sensitive, more specific

See "Predicting difficult intubation in apparently normal patients" Anesthesiology 2005;103:429-37. Figure 2.
Mallampati more sensitive than TMD
Mallampati less specific than TMD

However Fleischer book (Evidence Based Practice of Anesthesiology) pg 104 - seems equal sensitivity but TMD more specific (80-90% vs Mallampati 53-80%). go figure.

++ Just remember: specificity - more likely to diagnose people WITHOUT condition. A normal TMD is more likely to exclude difficult intubation. Alternatively, remember that MP has a high FALSE POSITIVE rate so is less SPECIFIC compared to TMD
-----------
++Rpt: Clinically the most significant murmur in pregnancy is?

A. MS
A. MS
-----------
++Rpt: Lap chole on citalopram. What is NOT relatively contraindicated?

A. Omeprazole
B. Clonidine
C. Pethidine
D. Tramadol
E. ?Midazolam
E. Midazolam (if it was an option!)

From: http://www.medscape.com/viewarticle/409573_4
Midazolam is metabolised by CYP3A4. Other SSRIs which inhibit CYP3A4 can increase concentration of midazolam.
--
A. Given the risk of dose dependent QT prolongation, citalopram dosage should not exceed 20mg/day when prescribed in combination with CYP450 2C19 inhibitors such as OMEPRAZOLE (drug interaction website); also from MIMS - 'Citalopram may have its serum concentration increased by omeprazole'.

B. BJA 2009, 102(4) 567-568 - Interaction between clonidine and escitalopram (similar interaction with citalopram): " This article showed that long-term treatment of rats with citalopram, among other antidepressants, caused a significant decrease in cerebral cortical binding of clonidine accompanied by a functional hyposensitivity of alpha2 adrenergic receptors. This led to attenuation of several central effects of clonidine such as hypothermia and sedation. This is the opposite of what we saw in our patient. It may be that there is a difference in the interaction of escitalopram with clonidine, as opposed to citalopram. Alternatively, this may represent an additive effect of the sedation seen with both drugs. Whatever the mechanism, caution is needed with all centrally acting drugs used in the critically ill.

C. Citalopram has its toxicity increased by Pethidine (MIMS)

D. Citalopram has its toxicity increased by Tramadol (MIMS)

-----------
++TMP-Jul10-036 [Aug10][Mar11][Sep11][Aug12]

Which is NOT a disadvantage of drawover vaporiser versus plenum vaporiser:

A. Temperature compensation (Basic temperature compensation)
B. Cannot use sevoflurane
C. Small volume reservoir
D. Flow compensation (Basic flow compensation)
E. ?

i.e. assuming what is NOT a disadvantage of the drawover (see wiki re: wording)
C. Small volume reservoir ?

or B. Cannot use sevoflurane? (u can but need two vaporisers - hence does this make B. 'false' and 'not' the answer???)
--
A. false - draw over vaporiser only has basic temperature compensation therefore performance is affected at extreme temperatures
b. false - can use sevoflurane HOWEVER output (max concentration) is limited so cannot induce with sevoflurane unless two vaporisers are used
c. small volume reservoir - true - "The fractional oxygen concentrate delivered to a patient is dependent on O2 output of the concentrator, MV of the pt and presence of the OET (oxygen economiser tube, aka reservoir). FiO2 conc is in depended of the ventilation pattern with the OET (reservoir) in place. Without an OET, performance is impaired and final FiO2 conc depends on flow of O2, MV and ventilation pattern. 1m length of tubing (internal vol 415ml) will produce an FiO2 of 30% with O2 at 1.0l/min and 60% with 4L/min. at normal MV. USING A LARGER INFLOW RESERVOIR CAN BE CUMBERSOME."
d. flow compensation - flow is determined by the patent
--
Basic principles behind draw-over vaporiser are same as for the plenum. However, draw-over (pullover) has a low resistance to flow and is relatively INEFFICIENT in comparison to plenum (pushover).

Plenum is used outside the circuit.

Draw-over may be used inside the breathing circuit, usually as part of a draw-over anaesthetic system. If used inside a circle breathing system, the expired vapour builds up to high concentration, hence close end tidal agent monitoring is recommended.

Fresh gas is drawn through the vaporiser because of a negative pressure generated downstream by the pt or ventilator. Flow is governed by pt's minute volume. Output varies with flow, decreasing as flow increases - calibration needs to cover a wide range of minute volumes (less accurate at high or low flows). Advantage is that they are portable and can be used where compressed gas is unavailable
--
-----------
1) Indication for IE prophylaxis in pt with CHD:

a) Gastroscopy with biopsy
b) Dental with RCT (?Root Canal Treatment?)
C) D&C
b) Dental with RCT
-
b) See table 2.13 eTG. Prophylaxis is always required for extraction, periodontal procedures including surgery, sub gingival scaling and root planing, replanting avulsed teeth, other surgical procedures. (essentially needed when gingival tissue is manipulated or the periodical region of teeth or perforation of oral mucosa)

a) and c) false - Table 2.15 eTG antibiotics.
Prophylaxis NOT required (low risk of bacteraemia)
- urethral catheterisation, uterine DILATATION AND CURETTAGE, sterilisation procedures, insertion or removal of IUDs
- vaginal delivery
- TOE
- ENDOSCOPY +/- BIOPSY, including colonoscopy
- PEG insertion

From eTG (therapeutic guidelines Australia)
- follows AHA lead, continuing a trend to reduce the categories of patients for whom prophylaxis is recommended while still specifying procedures for which prophylaxis is required.
Antibiotic prophylaxis is recommended for patients with cardiac conditions associated with the highest risk of adverse outcomes from endocarditis if undergoing a specified dental or other procedure. Cardiac conditions associated with highest risk of adverse outcomes from endocarditis:
- prosthetic cardiac valve or prosthetic material used for cardiac valve repair
- previous infective endocarditis
- CHD but only if it involves:
-- unrepaird cyanotic defects, including palliative shunts and conduits
-- completely repaired defects with prosthetic material or devices during first 6 months after procedure
-- repaired defects with residual defects at or adjacent to the site of a prosthetic patch or device
- cardiac transplantation with subsequent development of cardiac valvulopathy
- rheumatic heart disease in indigenous Australians only
-----------
41) Comparing New instrument of BP measurement with gold std - choice of test
Bland-Altman plot
-
From wikipedia
'Bland and Altman make the point that any two methods that are designed to measure the same parameter (or property) should have good correlation when a set of samples are chosen such that the property to be determined varies considerably. A high correlation for any two methods designed to measure the same property is thus in itself just a sign that one has chosen a wide spread sample. A high correlation does not automatically imply that there is good agreement between the two methods.

One primary application of the Bland Altman plot is to compare two clinical measurements that each provide some errors in their measure. it can also be used to compare a new measurement technique or method with a gold standard even so the interest of the Bland-Altman plot is contested in this particular case because the error pertains to the sole new measure.'
-----------
70) Most effective way to reduce renal failure in AAA surgery - Minimize cross clamp time
-----------
85) Drug must be given based on IBW -
-----------
94) Artery suppling anterior papillary muscle ( Pic)
-
Left ventricle has two papillary muscles which attach to the Mitral valve:
- Anterolateral (or anterior) - has a dual blood supply from the LAD and LCx.
- Posteromedial (or posterior) - blood supply from the posterior descending artery (which is a branch from RCA in 'right-dominant' circulations or LCx in 'left-dominant circulations).

Right Ventricle has 3 papillary muscles
- Anterior (the largest) attached to inferior border of the anterior wall
- Posterior located on the posterior wall
- Septal - small and often multiple.
-----------
105) Maintenance Fluid of choice for child on ventilator in ICU with closed head injury
-----------
109) Thoracic epidural helps with all except - circulating catecholamine levels
-----------
113) Low venous oxygen does not affect?
-----------
135) Time till cardiovesion can be comfortably done after new onset AF without ECHO?
a. 24 hrs
b. 48 hrs
c. 72 hrs
b. 48 hours
-----------
[Apr08][Oct08][Oct09][Aug10][?Aug12]
Central anticholinergic syndrome, which is NOT true:

A. Will improve with neostigmine

B. Peripheral anticholinergic symptoms

C. Caused by Anti-Parkinson drugs

D. CNS depression

E. Associated with agitation, delirium, and ???
A. Will improve with neostigmine
-
Neostigmine (quarternary ammonium structure) does not cross the BBB and will not have a central effect.

Physostigmine (tertiary ammonium structure) DOES cross the BBB and is the treatment of choice.



ACUTE ANTICHOLINERGIC SYNDROME

DEFINITION

Clinical syndrome resulting from antagonization of acetylcholine at the muscarinic receptor.

TOXIC CAUSES

Antihistamines (especially Promethazine, Trimeprazine,
Dimenhydrinate)
Antiparkinsonian drugs (e.g., Benztropine, Biperiden, Orphenadrine,
Procyclidine)
Antispasmodic agents (e.g., Clidinium, Glycopyrrolate,
Propantheline)
Belladonna alkaloids (e.g., Belladonna extract, Atropine, Hyoscine,
L-Hyoscyamine sulphate, Scopolamine hydrobromide)
Cyclic Antidepressants
Ophthalmic cycloplegics (e.g., Cyclopentolate, Homatropine,
Tropicamide)
Phenothiazines
Plants containing anticholinergic alkaloids (e.g., Atropa
belladonna, Brugmansia spp, Cestrum spp, Datura spp, Hyoscyamus
niger, Solanum spp). The tropane derivatives (alkaloids of
solanaceous plants and related drugs) are of greatest practical
importance.

CLINICAL FEATURES

The clinical diagnosis is based on the appearance of the
anticholinergic toxidrome. This toxidrome has central and
peripheral components:

The central anticholinergic signs and symptoms include altered
mental status, disorientation, incoherent speech, delirium,
hallucinations, agitation, violent behaviour, somnolence, coma,
central respiratory failure, and, rarely, seizures.

The peripheral anticholinergic syndrome includes hyperthermia,
mydriasis, dry mucosa membranes, dry, hot and red skin, peripheral
vasodilatation, tachycardia, diminished bowel motility (even
paralytic ileus), and urinary retention.

Rhabdomyolysis, cardiogenic shock or cardiorespiratory arrest may
occur exceptionally. Patients with closed-angle glaucoma may
suffer an acute precipitation of the condition. Patients with
benign prostatic hyperplasia are particularly prone to develop
urinary retention.

DIFFERENTIAL DIAGNOSIS

Alcohol withdrawal
Organic delirium (usually secondary to sepsis)
Psychiatric illness
Psychedelic drugs
Sympathomimetic drugs

RELEVANT INVESTIGATIONS

Measurement of blood and urine levels of the anticholinergic agents
are of little or no practical value.
Other laboratory examinations may be needed as dictated by the
general condition of the patient.

TREATMENT

Treatment is primarily supportive. The patient must be protected
from self-inflicted injury. This may require physical and/or
pharmacological restraint. Respiratory failure may require
intubation and controlled respiration. In cases of ingestion,
decontamination may be considered.

Diazepam: Administer 5 to 10 mg intravenously over 1 to 3
minutes. Repeat this dose as necessary to a maximal total dose of
30 mg.

The paediatric dose of diazepam is 0.25 to 0.4 mg/kg up to
maximal total dose of 5 mg in children up to 5-years-old and 10mg
in children over 5-years-old.

Physostigmine is a specific antidote for anticholinergic
poisoning and may be used under the following conditions :

1. Severe agitation or psychotic behaviour unresponsive to
other treatments.
2. Clinical evidence of both peripheral and central
anticholinergic syndrome.
3. No history of seizures.
4. Normal ECG, especially QRS width.
5. No history of ingestion or co-ingestion of tricylic
antidepressants or other drugs that delay
intraventricular conduction.
6. Cardio-respiratory monitoring in place and resuscitation
facilities available.

The dose of physostigmine is 1 to 2 mg (0.5 mg in children) by
intravenous injection over 2 to 5 minutes. If necessary, this dose
can be repeated after 40 minutes.

CLINICAL COURSE AND MONITORING

Complete recovery is expected over a period of hours to days.

In more severe cases of anticholinergic syndrome, cardiac rhythm
should be monitored and blood pressure frequently measured. Urine
output should be monitored so as not to overlook urinary retention.

-----------
[Aug08-138][Aug12] Ciliary ganglion

A sympathetic from inferior cervical ganglion

B located inferiorly within orbit

C may be damaged during a peribulbar block

D preganglionic parasympathetic supply from the supra trochlear nerve

E preganglionic parasympathetic originates from the Edinger Westpal nucleus
E preganglionic parasympathetic originates from the Edinger Westpal nucleus
-
Ciliary ganglion
- parasymp root - from Edinger Westphal part of oculomotor nucleus by a branch from the herve to the inferior oblique muscle from the inferior division of the oculomotor n.
- symp root - from superior cervical ganglion by branches of the internal carotid nerve
- sensory root - from a branch of the nasociliary nerve, with cell bodies in the trigeminal ganglion
- branches - short ciliary nerves to the eye
-----------
[Aug12] ++Rpt: Most effective treatment for post-sevoflurane agitation following grommets in a 4yo child?

A. 1 mg/kg propofol
B. 1 mcg/kg fentanyl
C. 1 mcg/kg clonidine
D. ?dose midazolam
E. Sucrose
A. 1mg/kg propofol
-
A. True. Dose varies. Dose 0.5-1mg/kg at the end of surgery.
B. Equally as effective as propofol however associated with more nausea and vomiting. 1mcg/kg fentanyl recommended dose. Intranasal dose is 2mcg/kg
C. false. Incorrect dose. 2-3mcg/kg clonidine after induction decreases agitation 0-10% as documented by Bock et al and kulka et al. - should also be given on induction?
D. false - according to practical approach to paediatric anaesthesia (via anzca) "Is not shown to CONSISTENTLY decrease incidence of EA; some studies suggest an improvement whereas others show no difference or an increase in the incidence of EA - dose of midazolam is 0.02mg/kg IV - but if paradoxical reaction then consider flumazenil 0.01mg/kg IV (max 0.2mg/dose) at 1-2 min intervals to a max dose of 1mg. In children over 12, give 0.2mg/dose at 1-2 min intervals to a total of 1mg.
-----------
[Aug12] 64) Peak incidence of vasospasm post SAH -
7-8 days
-
From UpToDate
'Vasospasm causes symptomatic ischaemia and infarction in ~20-30% of patients with aneurysmal SAH - it is the leading cause of death and disability after aneurysm rupture. It typically begins no earlier than day 3 after haemorrhage, reaching a peak at days seven to eight. The onset of clinical vasospasm is characterised by a decline in neurologic status, including the onset of focal neurologic abnormalities.'
-----------
[Aug12] 118) Absolute CI for ECT -

Increased ICP
Recent MI
Increased ICP - Clinical memorandum 12 RANZCP
--
From Clinical Memorandum #12 Royal Australian New Zealand College of Psychiatrists:
3.1 With the exception of raised intracranial pressure, there ar eno absolute contraindications to ECT…
Situations of high risk:
- Hypertension
- MI
- Bradyarrhythmias
- Cardiac pacemakers
- Intracranial pathology
- Aneurysms
- Epilepsy
- Osteoporosis
- Skull defect
- Retinal Detachment
- Concurrent medical illness
-----------
[Aug12] New: At what gestation should intraoperative monitoring of the fetus occur?

A. 20/40 weeks.
A.
-
From Chestnut's Ch17: 'Continuous FHR monitoring (using transabdominal Doppler ultrasonography) is feasible beginning at approximately 18-20 weeks gestation. However, technical problems may limit the use of continuous FHR monitoring between 18 and 22 weeks gestation. Transabdominal monitoring may not be possible during abdominal procedures or when the mother is very obese; use of transvaginal Doppler ultrasonography may be considered in selected cases.

FHR variability, which is typically a good indicator of feral well-being, is present by 25-27 weeks gestation. Changes in the baseline FHR and FHR variability caused by anaesthetic agents or other drugs must be distinguished from changes that result from feral hypoxia. Persistent severe feral bradycardia typically indicates true fetal compromise.

Intraop FHR monitoring requires someone who can interpret the tracing. A pal should be in place that addresses how to proceed in the event of persistent non reassuring feral status, including whether to perform emergency caesarean delivery. The greatest value of intraop FHR monitoring is that it allows for optimisation of the maternal condition if the fetes shows sign of compromise.'
-----------
[Aug12] New: Congenital prolonged QT syndrome treated with propranolol. How do you confirm an adequate response?

A. HR <60 /min
B. Decreased QT interval in response to a valsalva manoeuvre
C. Normal QT interval
Blunting of exercise heart rate (e.g. by >20%) or
Heart rate does not exceed 130bpm during exercise or
No change in QT interval in response to a Valsalva manoeuvre
-
From 'Long-QT Syndrome' NEJM 2008 Jan, 10.
"The mainstay of therapy for the long-QT syndrome has been beta-blockade. Long-acting preparations has been beta-blockade. Long-acting preparations such as nadolol and atenolol are usually used, and the efficacy of beta-blockade is assessed by blunting of the exercise heart rate (e.g. by >20%); beta blockers DO NOT SUBSTANTIALLY SHORTEN THE QT INTERVAL."

From 'Long QT Syndrome and anaesthesia' BJA 2003; 90:349-66
'Preoperatively, all patients with known LQTS should be on maintenance beta-blocker therapy, which myst be continued up to and including the day of surgery. Preop assessment of its adequacy should determine that the heart rate does not exceed 130/min during exercise; where exercise testing is impractical, there should ideally be no change in the QT interval in response to a valsalva manoeuvre in a fully beta blocked individual…

PPV should ensure that sustained high intrathoracic pressures are avoided as this MIMICS A VALSALVA which can PROLONG THE QT INTERVAL in patients who are not completely beta blocked.'
-----------
[Aug12] New: Endocarditis prophylaxis in patient with MVR appropriate for?

A. Dental procedure ?which
B. Rigid bronchoscopy
C. Upper endoscopy with biopsy
D. D&C
E. Lithotripsy
E. Lithotripsy
-
A. See table 2.13 eTG. Prophylaxis is always required for extraction, periodontal procedures including surgery, sub gingival scaling and root planing, replanting avulsed teeth, other surgical procedures. (essentially needed when gingival tissue is manipulated or the periodical region of teeth or perforation of oral mucosa)

B. false - Table 2.14 eTG antibiotics - Prophylaxis always required (high risk of bacteraemia) with any invasive procedure involving incision or biopsy of respiratory mucosa:
- tonsilloadenoidectomy
- rigid or flexible bronchoscopy WITH incision or biopsy
- surgery involving bronchial, sinus, nasal or middle ear mucosa including grommets.
Prophylaxis not required with RIGID BRONCHOSCOPY WITHOUT BIOPSY OR INCISION, endotracheal intubation.

C. and D. false - Table 2.15 eTG antibiotics.
Prophylaxis NOT required (low risk of bacteraemia)
- urethral catheterisation, uterine DILATATION AND CURETTAGE, sterilisation procedures, insertion or removal of IUDs
- vaginal delivery
- TOE
- ENDOSCOPY +/- BIOPSY, including colonoscopy
- PEG insertion

E. True - Table 2.15 eTG antibiotics
Prophylaxis always required (high risk of bacteraemia)
- any procedure where antibiotic prophylaxis is indicated for surgical reasons
- LITHOTRIPSY
- vaginal delivery with prolonged labour
- any genitourinary procedure in the presence of a GU infection unless already treating enterococci (for ELECTIVE cystoscopy or urinary tract manipulations, obtain a urine culture and treat any significant bacteriuria beforehand)
- any GI procedure in the presence of an intra-abdominal infection unless already treating enterococci

From eTG (therapeutic guidelines Australia)
- follows AHA lead, continuing a trend to reduce the categories of patients for whom prophylaxis is recommended while still specifying procedures for which prophylaxis is required.
Antibiotic prophylaxis is recommended for patients with cardiac conditions associated with the highest risk of adverse outcomes from endocarditis if undergoing a specified dental or other procedure. Cardiac conditions associated with highest risk of adverse outcomes from endocarditis:
- prosthetic cardiac valve or prosthetic material used for cardiac valve repair
- previous infective endocarditis
- CHD but only if it involves:
-- unrepaird cyanotic defects, including palliative shunts and conduits
-- completely repaired defects with prosthetic material or devices during first 6 months after procedure
-- repaired defects with residual defects at or adjacent to the site of a prosthetic patch or device
- cardiac transplantation with subsequent development of cardiac valvulopathy
- rheumatic heart disease in indigenous Australians only
-----------
[Aug12] New: Following an eclamptic seizure the dose of MgSO4 is?

A. 1 gram
B. 4 grams
B. 4 grams
-
From Anaesthesia 2012, 67, 1009-1020: 'The regimen recommended by the Collaborative Eclampsia Trial is 4-5g MgSO4 IV over 5 mins followed by an infusion of 1g/hr for 24 h. If recurrent seizures occur, an additional 2g IV MgSO4 should be administered.
-----------
[Aug12] New: In what proportion of people is the AV node supplied by the R coronary artery?
85-90%
-
From emedicine: 'In 85-90% of human hearts, the arterial supply to the AV node is a branch from the RCA that originates at the posterior intersection of the AV and the inter ventricular grooves (crux). In the remaining 10-15% of the hearts, a branch of the LCx provides the AV nodal artery. '
-----------
[Aug12] New: Incidence of headache in first week post-partum?
39%
-
From CEACCP 2011 Postpartum headache: diagnosis and management:
'Postpartum headache is described as a complaint of headache and neck or shoulder pain in the first 6 weeks after delivery. It is one of the most common symptoms with up to 39% of parturients experiencing headache in the first postpartum week.'
-----------
[Aug12] New: Oxycodone 20mg SR / Naloxone 20 mcg:

A. Decreased constipation
B. Reduced risk of drug misuse/abuse
A. Decreased constipation
--
From 2011 BJA 'An update on analgesics':
When taken by mouth, the naloxone is metabolised in the liver, so that peripheral antagonism of opioids exists in the guy, but that after metabolism, little naloxone passes in to the CNS. Hence the peripheral action of the naloxone when given with oxycodone is established… The aim is to prevent the GI side effects of opioids and various studies have demonstrated that this does work, with little effect on analgesia.
-----------
[Aug12] New: Oxycodone 20mg SR / Naloxone 20 mcg:

A. Decreased constipation
B. Reduced risk of drug misuse/abuse
A. Decreased constipation
-----------
[Aug12] New: Photograph of TOE transgastric SAX image of LV. Which artery supplies [anterior wall of LV arrowed] region?

A. LAD
B. RCA
C. PDA
D. LCx
A. LAD
-
LAD supplies anterior and anterolateral wall, apex, anterior IV septum and anterolat papillary muscle

RCA supplies RA, RV, SA and AV node

PDA (posterior descending artery) - branch of RCA - supplies inferior wall, posterior IV septum and posteromedial papillary muscle. in a few cases the LCx gives off the PDA

LCx - wraps around the left side to the posterior side of the heart. Supplies posterolateral side of the LV and gives off left marginal branches which also supply the LV
-----------
[Aug12] New: Regarding pyloric stenosis:
see also PP66

A. More common in females
B. Occurs most commonly in premmies
C. Acidic urine (or urinary acidification)
D. Cause of hypokalemia is vomiting (also remembered as hyperkalaemic metabolic alkalosis from GI losses)
C. Acidic urine (or urinary acidification)
-

Mostly male
Not more in premature
Hypokalaemia metabolic alkalosis
The excess loss of Cl depletes extracellular chloride and with the luminal loss of Hydrogen ions produces a metabolic alkalosis. The kidney tries to initially maintain blood pH by excreting an alkaline urine. HCO3 is excreted with Na and K until the overall volume deficit triggers an expansion of the extracellular volume rather than maintenance of pH. Na is resorbed, but K is lost via an aldosterone mediated mechanism and this leads to excretion of H ion resulting in "paradoxical aciduria" in an alkalotic patient.
-----------
[Aug12] New: Trauma patient. CXR (not given): air fluid levels adjacent to heart/diaphragm/ribs.

A. Ruptured diaphragm
B. Hiatus hernia
A. Ruptured diaphragm
--
Hiatal hernia typically appears as a round retrocardiac mass with air/fluid fluid level.

Ruptured diaphragm is associated with severe injury and occurs in 0.8-8% of patients after blunt trauma. It usually affects the left side. Suggestive findings include:
- irregularity of the diaphragmatic contour
- elevated hemidiaphragm in the absence of atelectasis
- contralateral shift of the mediastinum in the absence of the pleural effusion or pneumothorax
-----------
[Aug12] New: What drug should NOT be used for tocolysis in 32/40 female?

A. Indomethacin
B. Magnesium
C. Nifedipine
D. Salbutamol
B. Magnesium and
A. Indomethacin even ?D. salbutamol
-
B>A and D?

See Chestnut. Table 34-4.

Cochrane review on Magnesium in preterm labour (2002) found 'no evidence of a clinically important tocolytic effect for magnesium sulphate; it did not have any substantial effect of the proportion of women delivering within 48 hours, either overall, or in any subgroup analysis. Moreover, there was no evidence of any substantial improvements in neonatal morbidity. IN CONTRAST, MgSO4 was associated with an INCREASE IN FETAL AND PAEDIATRIC DEATHS. The higher death rate was present in the SUBGROUP where the maintenance dose of MgSO4, as in the study's protocol, was high, rather than low.'

Indomethacin can cause premature closure of ductus arteriosus in the fetus if given in 3rd trimester

Depends on reference. NSW health policy directive states that BEFORE 34/40 there are 5 classes of tocolytic agents available in Australia currently: CCB, Beta-agonists, nitric oxide donors, prostaglandin synthetase inhibitors and magnesium sulphate. The evidence to support the use of magnesium sulphate as a first line tocolytic is poor so it is not recommended.

The use of beta-agonists (like salbutamol) or multiple tocolytics is associated with a high incidence of serious adverse drug reactions. Both nitric oxide donors (like GTN) and prostaglandin synthetase inhibitors (like indomethacin) MAY HAVE A ROLE PRIOR TO 28 WEEKS.
-----------
[Aug12] NEW: What gestation to monitor uteroplacental flow

A 20 weeks

B 24 weeks

C 28 weeks

D 32 weeks

E 36 weeks
A 20 weeks
-
From Chestnut's Ch17: 'Continuous FHR monitoring (using transabdominal Doppler ultrasonography) is feasible beginning at approximately 18-20 weeks gestation. However, technical problems may limit the use of continuous FHR monitoring between 18 and 22 weeks gestation. Transabdominal monitoring may not be possible during abdominal procedures or when the mother is very obese; use of transvaginal Doppler ultrasonography may be considered in selected cases.

FHR variability, which is typically a good indicator of feral well-being, is present by 25-27 weeks gestation. Changes in the baseline FHR and FHR variability caused by anaesthetic agents or other drugs must be distinguished from changes that result from feral hypoxia. Persistent severe feral bradycardia typically indicates true fetal compromise.

Intraop FHR monitoring requires someone who can interpret the tracing. A pal should be in place that addresses how to proceed in the event of persistent non reassuring feral status, including whether to perform emergency caesarean delivery. The greatest value of intraop FHR monitoring is that it allows for optimisation of the maternal condition if the fetes shows sign of compromise.'
-----------
[Aug12] New: What is NOT a contraindication to MRI?
(See also AZ80 - different answers)

A. Pulmonary artery catheter
B. Arterial line
C. Scissors
D. Coiled ECG cable
E. Laryngoscope
B. Arterial line
--
Anaesthesia for MRI - PDF Aust Anaesth 2005
Invasive blood pressure transducers: These are not ferromagnetic, and are safe to use. Transducer cables should be kept out of the magnet bore, so as to avoid image distortion.
Intravenous cannulae needles: These are made from stainless steel and are safe.
Pulse oximeters: Standard pulse oximeters can malfunction in the MRI suite and have also been reported to cause patient burns due to overheating.
Standard ECG cables are insulated copper, and generate heat in the MRI scanner. This is especially so if the cables are allowed loop. Carbon fibre leads have less potential to heat.
Laryngoscopes: Although laryngoscopes may be non-ferrous, the batteries are highly ferromagnetic and cannot be used in close proximity to the magnet.
Pulmonary artery monitoring catheters and temporary
transvenous pacing leads contain nonferromagnetic
but electrically conductive material.
-----------
[Aug12] New: You see a patient in your clinic for a total knee replacement. He is 65 and has atrial fibrillation for which he takes dabigatran. He is otherwise well. A spinal anaesthetic is planned. What is the correct advice regarding his medication?

a. he should stop his dabigatran 7 days prior
b. he should stop his dabigatran 3 days prior
c. he should stop his dabigatran 3 days prior and have bridging enoxaparan
d. he should stop his dabigatran the day before and have an INR on the day of surgery
e. he should continue to take his dabigatran until the morning of surgery

(options from flashcard exchange)
b. he should stop his dabigatran 3 days prior
--
From Blood, 11 Oct 2012. Vol120 No. 15 "How I treat anti coagulated patients undergoing an elective procedure or surgery" -
In patients who are receiving dabigatran and require elective surgery, the timing of prep dabigatran interruption to ensure minimal or no residual anticoag effect depends on:
1. elimination half life of dabigatran
2. patient renal function and effect on dabigatran elimination
3. planned surgery and anaesthesia.

In pts with normal renal function (CrCl>80ml/min) or mildly impaired (50-80ml/min) dabigatran has elimination half life of 14-17 hours - there fore last dose should be given 3 days before surgery (i.e. skip 4 doses). This period of interruption corresponds to 4-5 half-lives (48-60 hours) which in turn corresponds to a minimal (~3-6%) anticoag effect at surgery.

In pts with moderately impaired renal function, half life is 16-18 hours - thus 4-5 half lives (54-90hrs) means last dose to be given 5 days before surgery (miss 8 doses).
-----------
[Aug12] Rpt: Best aspiration prophylaxis for urgent surgery?
see also TMP-113 worded differently.

A. Na Citrate
B. Ranitidine
C. Omeprazole
D. Metoclopramide
E. Cisapride
B. Ranitidine
or
?D. Metoclopramide
--
Previous question has said 'semi-urgent' surgery. Ranitidine takes 1-2 hours to take effect. Metoclopramide has onset 1-3 minutes. However not much evidence regarding metoclopramide. (See Interventions at c-section for reducing risk of aspiration pneumonitis Cochrane database 2010)
-----------
MN33b ANZCA version [2003-Aug] Q102, [2004-Apr] Q61, [2005-Sep] Q56, [Mar06][Apr08][Oct08][Aug12]

A multi-trauma patient opens his eyes and withdraws to painful stimuli, but does not respond to voice. He is moaning but makes no comprehensible sounds. His Glasgow Coma Score is

A. 5
B. 6
C. 7
D. 8
E. 9
D. GCS E2V2M4 =8
--
Eyes
1 - do not open
2 - open to pain
3 - open to voice
4 - open spontaneously
Voice
1 - no sounds
2 - incomprehensible sounds
3 - utters inappropriate words
4 - confused, disorientated
5 - orientated
Motor
1 - no movement
2 - extensor
3 - abnormal flexion
4 - flexion/withdrawal to pain
5 - localizes pain
6 - obeys commands
-----------
[Aug12] Rpt: Induction/LMA in ?kg child. Develops SVT. Most appropriate course of action?

A. Adenosine 100 mcg/kg
Depends on whether patient is haemodynamically stable.

from Australian Resus Council Guideline 12.5
- SVT is most common spontaneous onset dysrhythmia in childhood and infancy and may cause life-threatening hypotension.
- Usually re-entrant with rate of 220-300/min in infants, usually less in children
- QRS complex usually narrow and may be hard to differentiate from sinus tachycardia (ST)
- ST is variable with activity or stimulation, SVT is uniform and often sudden onset and offset.

If haemodynamically stable (adequate perfusion and blood pressure), initial tx of SVT
- vagal stimulation in the supine position
-- application to the face of a plastic bag filled with iced water or unilateral carotid sinus massage (infants, young children)
-- unilateral carotid sinus massage or if conscious ask to perform a valsalva (blow through a narrow straw) (older kids)
-- if mechanically ventilated, vagal stimulation may be effected by pharyngeal or tracheal suction.
- DO NOT perform eyeball pressure

Drug therapy:
- ADENOSINE is drug of CHOICE (Class A)
-- rapid IV or IO bolus and flush with 0.9% NaCl - 0.1 to 0.3mg/kg converts most cases to SR
-- initial recommended dose is 0.1mg/kg but if this is ineffective, the dose should be increased to 0.2mg/kg. (first dose should not exceed 6mg and second dose 12mg)
- Amiodarone may be used to treat haemodynamically stable or unstable SVT (Class A)
-- 5mg/kg over one hour then 5mcg/kg/min or in a schedule of 25mcg/kg/min for 4 hours then 5-15mcg/kg/min.
- Procainamide (15mg/kg) over 30-60 mins (Class B)
- AVOID Ca channel blockers

SVT may cause severe hypotension or pulselessness in which case synchronised DC shock should be given immediately 0.5-1.0 J/kg but increase to 2J/kg if needed.
-----------
[Aug12] Rpt: What is NOT a disadvantage of drawover vaporizer?

A. Basic temperature compensation
B. Basic flow compensation
-
A. false?
B. false?



Basic principles behind draw-over vaporiser are same as for the plenum. However, draw-over has a low resistance to flow and is relatively INEFFICIENT in comparison.
Plenum is used outside the circuit.
Draw-over may be used inside the breathing circuit, usually as part of a draw-over anaesthetic system. If used inside a circle breathing system, the expired vapour builds up to high concentration, hence close end tidal agent monitoring is recommended.
Fresh gas is drawn through the vaporiser because of a negative pressure generated downstream by the pt or ventilator. Flow is governed by pt's minute volume. Output varies with flow, decreasing as flow increases - calibration needs to cover a wide range of minute volumes (less accurate at high or low flows). Advantage is that they are portable and can be used where compressed gas is unavailable
-----------
[Aug] New: Bowel surgery patient. Best method for intraoperative optimization of fluid therapy?

A. Arterial pulse pressure contour analysis
B. CVP
C. PAOP
D. UO
??A. Arterial pulse pressure contour analysis
-----------
[Mar10][Aug12] You are performing a bronchoscopy, but are unsure of your location. Then you see trifurcation of bronchi. Most likely location is:
A. Right upper lobe
B. RML
C. RLL
D. LUL
E. Lingula <br
A. Right upper lobe
-
see wiki Mar10 exam
-----------
AM09e ANZCA version [Apr08] [Mar12-Q122][Aug12]

The diagnosis of neuroleptic malignant syndrome requires the presence of:

A. Diaphoresis
B. elevated plasma creatinine kinase (some recalled just ↑ CK)
C. hypertension
D. muscle rigidity
E. tachycardia
D. Muscle rigidity
and possibly B. ↑ CK if CK = creatine kinase NOT creatinine kinase (only using Levenson's crier for dx of NMS)
--
DSM IV-TR criteria: Severe muscle rigidity and elevated temperature associated with use of neuroleptic medication as well as two or more of the following -
- diaphoresis
- dysphagia
- tremor
- incontinence
- changes in level of consciousness ranging from confusion to coma
- mutism
- tachycardia
- elevated or labile BP
- leukocytosis
- laboratory evidence of muscle injury

Levenson's criteria (3 major, or 2 major and 4 minor criteria are needed for dx)
Major criteria
- fever
- rigidity
- elevated creatine kinase (CK)
Minor criteria
- tachycardia
- abnormal BP
- altered consciousness
- diaphoresis
- leukocytosis
-----------
[Mar12] - new
[Aug12]

Endocarditis prophylaxis

A Bicuspid valve
B Congenital repair > 12 months ago
C Rheumatic heart valve
D Uncorrected cyanotic heart disease
E MVP + ?MR
D. Uncorrected cyanotic heart disease
--
AHA ACC 2008 Guideline Update on Valvular Heart Disease: Focused Update on Infective Endocarditis Table 2
Class IIa
1. Prophylaxis against infective endocarditis is reasonable for the following patients at highest risk for adverse outcomes from infective endocarditis who undergo dental procedures that involve manipulation of either gingival tissue or the apical region of teeth or perforation of the oral mucosa:
- Patients with prosthetic cardiac valves or prosthetic material used for valve repair
- Patients with previous infective endocarditis
- Patients with CHD
-- Unrepairted cyanotic CHD, including palliative shunts or conduits
-- completely repaired congenital heart defect repaired with prosthetic material or device whether placed by surgery or catheter intervention, drive first 6 months after procedure
-- Repaired CHD with residual defects at the site or adjacent to the site of a prosthetic patch or prosthetic device (both of which inhibit endotheliazation)
- Cardiac transplant recipients with valve regurgitation due to a structurally abnormal valve
-----------
[Mar12][Aug12] NEW: 32 y/o male. Weakness distal and prox muscles, infection 10 days ago, no sensory involvement, temp 37.8, facial weakness. Cause:
see MZ69

A. Guillain Barre
B. Myasthenia Gravis
D. Poliomyelitis
E. ?Acute encephalitis
F. ?Polymyositis
A. Guillain Barre
or
D. Poliomyelitis
--
A. Guillain Barre - clinical picture states "NO sensory involvement" though GBS does present with some sensory symptoms
"The classical picture is that of ascending limb weakness with areflexia, although a purely sensory variants has been well documented" - CEACCP 2003 article.
From Emedicine:
The classic description of GBS is that of a demyelinating neuropathy with ascending weakness, many clinical variants have been well documented in the medical literature. Acute inflammatory demyelinating polyradiculoneuropathy (AIDP) is the most widely recognised form in western countries, but variants known as acute motor axonal neuropathy (AMAN) and acute motor-sensory axonal neuropathy are also well recognised (AMSAN).

Hx - typical GBS patient (which in most cases will be AIDP) present 2-4 weeks post benign resp or gastro illness with complaints of finger dysethesias and proximal weakness of LL. Weakness may progress over hours to days to involve arms, truncal muscles, cranial nerves and muscles of respiration.

Weakness - classic clinical picture is ascending and symmetrical. Lower limbs usually involved before upper. Proximal before distal. Trunk, bulbar, and resp muscles can be affected also. Facial and oropharyngeal weakness usually appears after trunk and limbs are affected. Miller-Fisher variant is unique in that this subtype begins with CN deficits.

Sensory - Most pt complain of parenthesis, numbness or similar sensory changes - these often precede weakness. Generally start in toes and fingertips, progressing upwards but not extending beyond wrists or ankles. Loss of vibration, proprioception, touch and pain distally may be present. Symptoms are usually mild. In most cases, objective findings of sensory loss tend to be minimal and variable.

B. Myasthenia Gravis - unlikely; usually presents with ocular symptoms rather than distal/prox muscle weakness. Variable in presentation, but no sensory component.

C. Poliomyelitis - story fits best. From emed - Most patients are asymptomatic or have mild systemic symptoms. Only 5% exhibit different severities of nervous system involvement from non paralytic poliomyelitis to the most severe form of paralytic poliomyelitis.

Nonparalytic or preparalytic poliomyelitis
- prodromal symptoms include generalised, non throbbing headache; fever 38-40 deg; sore throat; anorexia; NV; muscle aches which may last 1-2 wks.
- headache, fever and signs of nervous system involvement
- may develop into paralytic polio
Paralytic
- incubation period from firus exposure to neurologic phase can last 4-10DAYS but may extend 4-5 wks
- severe muscle pain and spasms followed by weakness develop. Muscles weakness maximal within 48 hrs but may develop for longer than a week. No progression of weakness should be noted after temperature drops to normal for 48 hours. Weakness is asymmetric LL>UL
- Muscle tone is flaccid, reflexes initially brisk then absent. Transient occas persistent coarse fasciculations are also seen
- Pts COMPLAIN of parenthesis in affected limbs WITHOUT real sensation loss
- paralysis remains for days or weeks before slow recovery

Paralytic polio with bulbar involvement
- bulbar paralysis with spinal involvement more common in adults, most often involves medulla leading to dysphagia, dysphonia, resp failure and vasomotor disturbance.
- Cranial nerve involvement in 10-15% of cases (including facial nerve!!)
-----------
[Mar12][Aug12] NEW: Blood flow across which of the following is used to estimate pulmonary artery pressures during echocardiography?

A. Tricuspid valve

B. Pulmonary valve

C. Mitral Valve
A. Tricuspid valve
--
But need TR. see online textbook on TOE.
-----------
[Mar12][Aug12] New: Increased risk of post-partum haemorrhage in:

A. Nulliparous patient
B. Patient < 20 years old
C. Factor V Leiden deficiency
D. Oligohydramnios
E. Prolonged labour
E. Prolonged labour
--
A. false. Grand multiparty WAS traditionally thought to be a risk factor for PPH - large observational study in Australia "Is grand multiparty an independent predictor of pregnancy risk? A retrospective observational study" MJA 2003 negated this. No info re: nulliparity but unlikely
B. flase. From 'A Textbook of Postpartum Hemorrhage' (extract via google books): INCREASING maternal age appears to be an independent risk factor for PPH.
C. false. Factor V Leiden deficiency DOES NOT exist.
Factor V Leiden - also known as Activated protein C resistance - is a disorder where the Leiden variant of factor V cannot be inactivated by activated protein C. It is the most common hereditary HYPERcoagulability disorder amongst Eurasians. Normally factor V acts as a cofactor to allow factor Xa to activate thrombin which cleaves fibrinogen to fibrin -> clot. aPC limits the extent of clotting by cleaving and degrading factor V.
Factor V deficiency is a rare inherited coagulopathy also known as Owren disease or parahemophilia.
D. false - POLYhydramnios is a risk factor as it is associated with uterine atony. Could not find anything implicating oligohydramnios.
-----------
[Mar12][Aug12] New: Young child with WPW undergoes general anaesthesia. Intra operatively developed tachycardia. HR 220, BP 80/40. Best drug to cardiovert

A. Adenosine
B. Amiodarone
C.
A. Adenosine
--
from Australian Resus Council Guideline 12.5
- SVT is most common spontaneous onset dysrhythmia in childhood and infancy and may cause life-threatening hypotension.
- Usually re-entrant with rate of 220-300/min in infants, usually less in children
- QRS complex usually narrow and may be hard to differentiate from sinus tachycardia (ST)
- ST is variable with activity or stimulation, SVT is uniform and often sudden onset and offset.

If haemodynamically stable (adequate perfusion and blood pressure), initial tx of SVT
- vagal stimulation in the supine position
-- application to the face of a plastic bag filled with iced water or unilateral carotid sinus massage (infants, young children)
-- unilateral carotid sinus massage or if conscious ask to perform a valsalva (blow through a narrow straw) (older kids)
-- if mechanically ventilated, vagal stimulation may be effected by pharyngeal or tracheal suction.
- DO NOT perform eyeball pressure

Drug therapy:
- ADENOSINE is drug of CHOICE (Class A)
-- rapid IV or IO bolus and flush with 0.9% NaCl - 0.1 to 0.3mg/kg converts most cases to SR
-- initial recommended dose is 0.1mg/kg but if this is ineffective, the dose should be increased to 0.2mg/kg. (first dose should not exceed 6mg and second dose 12mg)
- Amiodarone may be used to treat haemodynamically stable or unstable SVT (Class A)
-- 5mg/kg over one hour then 5mcg/kg/min or in a schedule of 25mcg/kg/min for 4 hours then 5-15mcg/kg/min.
- Procainamide (15mg/kg) over 30-60 mins (Class B)
- AVOID Ca channel blockers

SVT may cause severe hypotension or pulselessness in which case synchronised DC shock should be given immediately 0.5-1.0 J/kg but increase to 2J/kg if needed.
-----------
[Mar12][Aug12] REPEAT: Air bubble in arterial line. Causes decreased:

A Damping
B Resonant freq of system
B
-----------
[Mar12][Aug12] What is the best indicator of pending respiratory depression when using a morphine PCA?
see PN42

a. Respiratory rate
b. Sedation score
c. Reduced saturations
d. HR
b. Sedation score
-----------
[Mar12][Aug12] You see a man in his 60s in clinic 1 week prior to laparoscopic cholecystectomy. He has dilated cardiomyopathy with an ejection fraction of 30%, but does not get dyspnoeic with normal activities of daily living. What is the most appropriate management of his heart failure?
See also TMP-Sep11-056 but different options.

a. Frusemide
b. Amiodarone
c. ACE (I presume ACE inhibitor…)
d. Digoxin
e. Biventricular pacemaker
f. stop beta blocker (option in Aug12)
c. ACEI
--
a. Frusemide - false; indicated if he had stage B failure which is past or current symptoms of HF
b. Amiodarone - false; should be avoided where possible. "Amiodarone should not be considered as part of the routine treatment of patients with HF, with or without frequent PVCs or asymp non sustained VT; HOWEVER it remains the agent most likely to be safe and effective when anti arrhythmic tx is needed to prevent recurrent AF or symptomatic ventricular arrhythmias" (2009 HF guidelines)
d. Digoxin - false; Class III recommendation for stage B HF (i.e. nor recommended at all). See section in guidelines re: digoxin (essentially can improve symptoms but not primary Tx with acute exacerbation)
e. Biventricular pacemaker - false; not first line tx. class IIa or IIb recommendation.
--
NYHA I (no limitation in ordinary physical activity) and Stage B (structural heart disease but without signs or symptoms of HF). ACC AHA guidelines on heart failure 4.2
Class I recommendations
- Beta blockers and ACEI should be used in all patients with a recent or remote history of MI regardless of EF or presence of HF
- Beta blockers (level C evidence) are ACEI (level A) should be used in all patients with a reduced LVEF with no symptoms of HF even if they have not had an MI
- ARB should be started to post-MI patients without HF who are intolerant of ACEI and have a low EF
- Patients who have not developed HF symptoms:
-- should be treated according to contemporary guidelines post acute MI;
-- coronary revascularization is recommend in appropriate patients as per guidelines (in Mx of chronic stable angina)
-- valve replacement or repair as per guidelines

If he has had prior symptoms of HF it is Stage C heart failure in which case Frusemide is also indicated
-----------
[May09][Aug12] Young woman with subarachnoid haemorrhage, hyponatraemia and increased urinary sodium (did not specify if high sodium concentration or total amount lost). What is likely cause?

A. cerebral salt wasting syndrome

B. SIADH

C. HHH therapy

D. Excess NS administration

E. diabetes insipidus
A or B, depending on volume status
--
CSWS is usually caused by brain injury/trauma or cerebral lesion, tumor, or hematoma. CSWS is a diagnosis of exclusion and may be difficult to distinguish from the syndrome of inappropriate antidiuretic hormone (SIADH), which develops under similar circumstances and also presents with hyponatremia. The main clinical difference is that of total fluid status of the patient: CSWS leads to a relative or overt hypovolemia whereas SIADH is consistent with a normal to hypervolemic range. Random urine sodium concentrations tend to be lower than 100 mEq/L in CSWS and greater in SIADH . If blood-sodium levels increase when fluids are restricted, SIADH is more likely.
-----------
[May09][Mar12][Aug12]

2 yo 15kg child following seizure on surgical ward. Admitted with appendicitis and perforation. 60ml/hr of ½ N. Saline 5% dextrose

Na+ 119
K+ 4.5
HCO3- 19
Cl- 90

Best treatment would be

A. Desmopressin
B. Frusemide
C. 3% saline (hypertonic)
D. Normal Saline
E. Fluid restrict

(some stems have give phenytoin)
C. 3% Saline (or 3% NaCl - NOT 3% normal saline)
--
ref: http://www.rch.org.au/clinicalguide/guideline_index/Hyponatraemia/

Main causes of hypoNa in children are
- administration of hypotonic fluids (IV or enteral)
- conditions with impaired free H2O excretion and high ADH levels
- Relative excess fluid intake in a child receiving exogenous anti-diuretic agents (e.g. Desmopressin for nocturnal enuresis)
- GI fluid losses
Less common but more important causes are:
- adrenal insufficiency
- defect in renal tubular absorption, including obstructive uropathy
- psychogenic polydipsia

Management
- Ideal rate of Na correction depends on presence and severity of symptoms. Rapid correction (>8mmol/L Na+/24h) can result in cerebral demyelination - especially if hypoNa has been present for >5 days and rapidly corrected.
In children with seizures or CNS depression
- Notify ICU
- ABC resus and IV anticonvulsants as indicated. HypoNa seizures often respond poorly to conventional anticonvulsants and Na correction should not be delayed. The Na should be raised until it reaches 125mmol/L or until seizures stop - whichever occurs first
- Use IV 356aCl solution - 4ml/kg over 15-30 minutes. This will raise serum Na by 3mmol/L and will usually stop the seizures. Give thru a CVC where possible but do not delay administration in a fitting child to insert the CVC.
- Measure serum Na after first bolus. Ongoing seizures and persistent hypoNa will require more 3% NaCl
- Many children with hypoNa and seizures will have other reasons for seizures (fever, meningitis, hypoglycaemia) and these should be addressed
- After the seizures have resolved the total sodium correction (including the bolus) should not exceed 8mmol/L per day (e.g. from 122-130mmol/L)
- Measure electrolytes every 2 hours until stable, then every 4-6 hours until the serim sodium is normal and the child is off IV fluids.

If the child has no symptoms of hypoNa
- Management depends on volume status and active correction with 3% NaCl is not needed.

-----------
AA04 - ANZCA version [2004-Apr] Q72, [2004-Aug] Q82 [Oct08][Oct09][Mar10][Aug10][?Aug12]

Histamine release during anaphylaxis does NOT cause:

A. tachycardia

B. decreased myocardial contractility

C. coronary vasodilatation

D. a shift in the pacemaker site

E. an increased PR interval

F. decreased pulse conduction
B. decreased myocardial contractility



Stoelting p.386
-----------
AB56 [Apr07][Jul07][Mar12][Aug12]
The most commonly reported cause of death after blood transfusion

A. TRALI

B. GVHD

C. Incompatability

D. Anaphylaxis

E. Bacterial contamination
A. TRALI

see long-winded wiki discussion
-----------
AC62b [Apr07][Sep11][Aug12]

The most commonly reported cause of awareness during general anaesthesia for a non-obstetric procedure is

A. inadequate intra-operative opiate analgesia

B. equipment failure

C. the use of total intravenous anaesthesia

D. faulty anaesthetic technique

E. use of a laryngeal mask airway
D. faulty anaesthetic technique


-----------
AC111 [?Aug12]

Acute visual loss after nonocular surgery is most commonly caused by

A. ischaemic optic neuropathy

B. prolonged direct compression of the globe

C. cortical blindness

D. retinal artery occlusion

E. electrolyte imbalance
A. ischaemic optic neuropathy



-----------
AC111c ANZCA version [Aug04][Apr05][Aug12]

The most common diagnosis of the cause of blindness following anaesthesia in the prone position is (or spinal surgery):

A. drug effect

B. emboli

C. globe compression

D. ischaemic optic atrophy

E. psychiatric disturbance
D. ischaemic optic atrophy


-----------
AC159 [Mar11][Sep11][Mar12][Aug12]

A patient has a seizure when returned to the ward after a Carotid endarterectomy (CEA). BP has been hard to control. What to do to prevent further seizures?

A. Add another antihypertensive

B. Start antiplatelet drugs

C. Start anticonvulsants

D. Do angiography and stent

E. Nimodipine
A. Add another antihypertensive??

--
Cerebral hyper perfusion syndrome - triad of ipsilateral migraine-like headache, seizure, and transient focal neurologic deficits in the absence of cerebral ischaemia.
-----------
AC?? [May09] [Oct09] [Mar10] [?Aug12]

ASA grading was introduced to

A. Predict intraoperative anaesthetic risk

B. Predict intraoperative anaesthetic and surgical risk

C. Standardise physical status classification of patients

D. Predict perioperative anaesthetic risk

E. Predict perioperative anaesthetic and surgical risk
C. Standardise physical status classification of patients

-----------
AM25 [Apr08] [Oct08] [Sep11] [?Aug12]

In an acute malignant hyperthermia episode

A. the serum creatine kinase level peaks within one hour

B. the peak serum creatinine kinase level is a good indicator of the amount of muscle involved

C. elevated creatine kinase levels contribute to acute renal failure

D. the serum myoglobin level does NOT peak for at LEAST 24 hours

E. muscle rigidity occurs in 75% of cases
E. muscle rigidity occurs in 75% of cases
--
A - FALSE

creatine kinase peaks around 24 Hrs?

B - FALSE

peak serum CK is not a reliable indicator of MH vs non-MH, as CK levels of MH patients often within levels expected from surgery itself..."patients who have had an acute MH episode during a surgical procedure may have peak CK values within the range of CK values expected from the procedure itself"...Creatine kinase alterations after acute malignant hyperthermia episodes and common surgical procedures, JF Antognini, Anesth Analg 1995 (81) 1039-42 A&A article...drstitch
Serum myoglobin (by radioimmunoassay) and creatine kinase were measured for up to 7 days in 30 patients following surgical procedures, including total hip replacement and bilateral subcostal abdominal incisions. Serum myoglobin reached a maximum of 1390 µg litre–1 (median 345 µg litre–1 for major surgery patients) on the first postoperative day but levels were still elevated by day 7 in some patients. Creatine kinase reached a maximum of 1339 i.u. litre–1 at day 2 (median 422 i.u. litre–1 for major surgery patients), generally peaking 1 day after myoglobin in individual patients. These values may have significance when investigating a suspicion of coincident perioperative events such as myocardial infarction or malignant hyperthermia...Serum myoglobin and creatine kinase following surgery, A. S. Laurence, British Journal of Anaesthesia, 2000; Vol. 84, No. 6 763-766

C - FALSE

Myoglobin

D - FALSE

myoglobin peaks ~ 6hrs later

E - TRUE

rigidity 80%


-----------
AM41 ANZCA version [2004-Aug] Q15, [Mar06] Q11, [Jul07][Mar10][Sep11][?Aug12]

The most frequently reported clinical sign in malignant hyperpyrexia is

A. arrhythmia

B. cyanosis

C. sweating

D. tachycardia

E. rigidity
A. Tachycardia
--
From Yao Ch53
Specific clinical features for MH
- Increase in end-tidal CO2 during constant ventilation (the most sensitive and specific sign)
- Generized rigidity (extremely high specificity)
- Masseter Muscle Rigidity (MMR)
- Increased temperature (not uncommonly higher than 40 degC)
Nonspecific
- TACHYCARDIA (earliest and most consistent clinical sign however not specific)
- Tachypnoea
- Arrhythmias
- Skin Mottling
- Profuse sweating
- Altered BP


-----------
AT20c [Jul07][Apr08][Aug12]

All of the following are major complications of mediastinoscopy EXCEPT:

A. Cardiac laceration

B. Air embolism

C. Pneumothorax

D. Major haemorrhage

E. Recurrent laryngeal nerve damage
A. Cardiac laceration


Continuing Education in Anaesthesia, Critical Care & Pain | Volume 7 Number 1 2007
-----------
AT27 [Apr07][Mar11][Sep11][Mar12][?Aug12]

Following a left sided pneumonectomy, a left intercostal drain is placed and connected to an underwater drainage system. In the postoperative period

A. A leakage of air is expected from the drain

B. The patient should be nursed in the right lateral decubitus position

C. The underwater seal drain should be left on continuous free drainage

D. The underwater seal drain should be left on continuous free drainage and connected to wall suction for 5 minutes every hour

E. The underwater seal drain should remained clamped and be released for a short period every hour
E. The underwater seal drain should remained clamped and be released for a short period every hour
--

B is also correct - but E is more correct. If signs of cardiac herniation occur then the patient needs to be nursed operative side up. see wiki.

NEVER suction drain after pneumonectomy (may cause cardiac herniation), leave CLAMPED with intermittent unclamping.

Cardiac Herniation: If the pericardium has been disrupted following pneumonectomy, herniation of the heart may occur through the defect. Acute angulation of the heart and great vessels may produce arrhythmias or severe hypotension in the presence of high filling pressures.The patient should be immediately turned with the non-operative lung dependent. If the chest drain on the operative side had mistakenly put on suction, it should be opened to air. Immediate reoperation may be required.
-----------
AZ03 [Jul07][Apr08][Aug12]

The BEST indication of a difficult intubation in morbid obesity:

A. Mallampatti Score

B. Neck circumference

C. Limited neck movement

D. TMD

E. Body weight
B. Neck circumference


JB Brodsky, HJM Lemmens, JG Brock-Utne, MVierra, LJ Saidman, Morbid Obesity and Tracheal Intubation. Anesth Analg 2002;94:732–6

➮ Factors looked at included: "height, weight, neck circumference, width of mouth opening, sternomental distance, thyromental distance and Mallampati score"

➮ "Logistic regression identified neck circumference as the best single predictor of problematic intubation. Mallampati score inclusion did not further improve the model in our limited study with only 12 problematic intubations. In patients with a large neck, the view during direct laryngoscopy was poorer."



-----------
AZ80 [Apr08][Mar10][Aug10][Aug12]

Which of the following is NOT an absolute contraindication to magnetic resonance imaging?

A. cardiac pacemaker

B. cerebral aneurysm clips

C. cochlear implant

D. implanted defibrillator

E. prosthetic heart valve
E. prosthetic heart valve

see wiki debate.

http://bitc.bme.emory.edu/MRI_contraindications.html

Absolute Contraindications
- electronically, magnetically, and mechanically activated implants:
- cardiac pacemakers
- ferromagnetic or electronically operated stapedial implants
- hemostatic clips (CNS)
- metallic splinters in the orbit

Relative Contraindications
- electronically, magnetically, and mechanically activated implants:
- other pacemakers, e.g.,
• for the carotid sinus;
• insulin pumps and nerve stimulators;
• lead wires or similar wires
- non-ferromagnetic stapedial implants
- cochlear implants
- prosthetic heart valves (in high fields, if dehiscence is suspected)
- hemostatic clips (body)
- makeup and tattoos
- congestive heart failure
- pregnancy (claustrophobia)

--

From Olive 2005 MRI ANZCA 'Blue Book':

a) Pacemakers: A permanent pacemaker constitutes a contra-indication to MRI scanning. Currents generated in the pacemaker circuitry, even at low field strengths (e.g. 17 Gauss), can cause serious malfunctions.10 This has resulted in at least one death in Australia. Reports exist of patients with pacemakers being scanned in low field strength scanners (0.5T),11 but the risk-benefit ratio needs to be seriously considered.

b) Cochlear implants: These devices contain a magnet that holds the external component to the subcutaneous receiver and is also involved in signal transmission. This magnet may move and cause injury.12 If it is known at the time of cochlear implant insertion that future MRIs will be required, the internal magnet can be omitted and the external component secured with adhesive patches. In such a case, an MRI could be performed, although significant artefact could be expected in head scans.

c) Orthopaedic prostheses: The metal components of these prostheses are generally titanium or chromium/cobalt. Screws and plates are stainless steel. While these implants may cause some image artefact, they are safe in the MRI scanner. External fixation devices often contain iron, so are contraindicated.

d) Prosthetic heart valves: These valves and annuloplasty rings undergo minimal heating and torque and are safe in MRI,13, 14 although some artefact may be caused.

e) Aneurysm clips: These are variable. Earlier models were ferromagnetic, so could move in the magnetic field, with potentially disastrous consequences. Most modern clips are non-ferromagnetic and are safe in MRI.15 Due to this variability, manufacturers’ specifications should be checked.
-----------
CTG [pictured] demonstrating late decelerations. Most likely cause?

A. Fetal asphyxia
B. Head compression
C. Cord compression
D. Uteroplacental insufficiency
D. Uteroplacental insufficiency
-----------
EM16b ANZCA version [2002-Mar] Q68, [2002-Aug] Q64, [2005-Apr] Q94, [2005-Sep] [Apr08] [Sep11] [Mar12] [Aug12]

Circuit disconnection during spontaneous breathing anaesthesia

A. will be reliably detected by a fall in end-tidal carbon dioxide concentration

B. will be detected early by the low inspired oxygen alarm

C. will be most reliably detected by spirometry with minute volume alarms

D. may be detected by an unexpected drop in end-tidal volatile anaesthetic agent concentration

E. can be prevented by using new, single-use tubing
D. may be detected by an unexpected drop in end-tidal volatile anaesthetic agent concentration


CJA 48:847-849 (2001)
A breathing circuit disconnection detected by anaesthetic agent monitoring
-----------
EM37 [Sep11][Mar12][Aug12]

Systemic vascular resistance index (SVRI) is calculated from

A. systemic vascular resistance multiplied by body surface area

B. systemic vascular resistance divided by body surface area

C. mean aortic and central venous pressure difference divided by cardiac output

D. cardiac index divided by the mean aortic and central venous pressure difference

E. none of the above
A. systemic vascular resistance multiplied by body surface area


SVR = ((MAP - CVP) *80)/CO

CI = CO / BSA

SVRI calculated by substituting CI for CO in the equation for SVR

ie. SVRI = ((MAP - CVP) / CI


-----------
EM39 [Apr03][Mar12][Aug12]

The presence of small bubbles of air in an invasive arterial pressure monitoring system will always decrease the

A. damping coefficient of the system

B. resonant frequency of the system

C. recorded systolic pressure

D. recorded mean pressure

E. extinction coefficient of the system
B. resonant frequency of the system


Bubbles
➭ always increase the damping coefficient
➭ always decrease the resonant frequency
➭ may cause a lower recorded systolic pressure
➭ never alter the recorded mean pressure (only dependent on static accuracy)
➭ an extinction coefficient is the fraction of light lost to scattering and absorbtion per unit distance in a participating medium - nothing to do with transducers!
-----------
EM66 [Apr09][Oct09][Mar2010][Aug2010][?Aug12]

What's the most appropriate mode for neuromuscular monitoring during aneurysm clipping?

A. TOF count
B. TOF ratio
C. Post tetanic count
D. DBS
E. ?
C. Post tetanic count (PTC)
-
Used to assess profound block
-----------
EZ79 [May09][Mar12] [Aug12]

An infant is anaesthetised and ventilated using an endotracheal tube and circle breathing system with C02 absorber. The item which causes the most resistance to breathing is the

A. airway pressure limiting (APL) valve

B. circuit hosing

C. endotracheal tube

D. heat and moisture exchange filter

E. inspiratory and expiratory valves
C. endotracheal tube
-----------
EZ85 ANZCA version [Apr07] [Jul07] [Aug12]

In a rotameter the:

A. Bobbin spins inside a tube that has parallel sides

B. Flow is laminar at high flow rates

C. Height of the bobbin is proportional to the pressure drop across the bobbin

D. Pressure drop across the bobbin is constant at varying flows

E. Resistance increases with increasing gas flow
D. Pressure drop across the bobbin is constant at varying flows - true:

"The Rotameter and the Wright peak flow meter are examples of variable orifice flowmeters. Constant pressure in the former is provided by the pressure of the bobbin; the pressure is the ratio of the product of the mass and gravity over the cross sectional area of the bobbin (p = F/A = m a/A). The Rotameter is tapered, and thus the magnitude of the orifice around the bobbin corresponds to the magnitude of gas flow." (Basic science for anaesthetists).

From AnaesthesiaUK website:

"Rotameters are widely used in gas delivery systems for continuous flow measurement. Several designs have been used, though now almost all use a vertical tapered tube containing a bobbin or ball which is supported by the gas flow as it passes upwards through the tube.
The weight of the bobbin (and thus the pressure drop required to support it) is constant, but as the flow increases its position in the tube rises, lowering the resistance as a larger pathway is created alongside the bobbin.

Amalgamation of the space for gas flow around the bobbin to equate to a simple pathway reveals that at the bottom of the flowmeter the resulting dimensions lead to laminar flow but at the top of the tube produce turbulent flow. The physical characteristics of the gas that determine the resulting flow are therefore viscosity at the bottom and density at the top of the tube. The main implication of this is that calibration of rotameters is gas-specific and for accuracy its use must be restricted to that gas. Calibration is usually done to read the flow rate from the top of a bobbin but the centre of a ball.

The range of gas flow measurements can be increased by using two tubes (one for low and one for high flow rates), or by varying the taper so that a greatly increased diameter results at the top of the tube.

Inaccuracy results from anything that causes the bobbin to stick in the tube, including dirt or static electricity. To prevent build-up of static, the inside walls of a rotameter and its mounting points are made of conductive material. To demonstrate that the bobbin is not stuck, it has angled flutes to produce rotation, which is made easier to see by appropriate colouring.

Back pressure caused by downstream resistance also leads to an inaccurately low reading on a rotameter, though the actual flow is the same as that shown before the resistance was applied."
-----------
EZ93 [Sep11][Aug12]

What is the chemical used in soda lime to indicate exhaustion
A. ethyl violet
B. potassium permanganate
C. ?
A. Ethyl violet

Ethylviolet is a pH-sensitive triarylmethane dye used as an indicator in CO2-absorbents. It changes from colorless to blue/violet.
-----------
EZ94 (Mar12)[Aug12]

A home handyman leaves his electricity turned on whilst fiddling with wires [repairing a power outlet]. He has a RCD. What happens if he touches the neutral and ground wires?

A. Nothing will happen
B. Receives macroshock
C. Protected from macroshock by RCD
D. Protected from microshock by domestic fuse
E Receives microshock
A. Nothing will happen
- For current to flow, there must be a complete circuit from one voltage terminal (LIVE) via the body to the other terminal (NEUTRAL). The GROUND or earth wire returns leaks from the equipment back via the earth (instead of anyone who may touch it).
Theoretically nothing should happen if he was to touch the neutral and ground wires.

B. Receives a macroshock - this would be the case if he touched the active and neutral simultaneously.
--
From 'Electrical hazards causes and prevention' Anaes Int Care Med:
A residual current device is much more sensitive and can break a circuit before a person perceives any pain. This relies on the principle that normally in a correctly working system there is a balance of current in the live and neutral leads. In a fault situation, there is an imbalance of current and this causes an electrically operated switch to break the circuit.

HOWEVER - if he touches active an neutral wire AT THE SAME TIME - no difference in current is detected and the current will pass through him causing a shock.

From http://www.allstatesafety.com.au/services/rcd-residual-current-device-safety-switch/
What an RCD will not protect;
If a user was to touch both active and neutral at the same time, then current flow would be even through both circuits. RCD WON’T TRIP
Scenario #1 A carpenter using a tool with an internal fault, can be electrocuted from a positive to neutral connection, because both positive and neutral would be feeding electricity evenly into the body, hence the RCD wouldn’t trip.
Scenario #2 An ice machine with a faulty earth and a positive short to it metal frame is touched by a user; positive could flow into the users hands join the neutral side within a fan or compressor. This would happen because an earth wasn’t present. Electrons always take the path of least resistance. If a good earth contact was present then as soon as a positive short was made to the frame positive would have gone to earth and its amperage would have been higher than the negative, therefore the RCD would have tripped.
In basic terms if you short;

• Positive to earth – RCD Trips
• Positive to negative with a good earth – RCD Trips
• Positive to negative – RCD wont trip
• Positive to a metal frame with no earth – RCD wont trip
-----------
Half life of mast cell tryptase?
(see also AA23 or AA13)

A. 1 hour
B. 2 hours
b. 2 hours
--
In vivo half life of tryptase is 2.5 hours
-----------
IC67 [Mar12][Aug12]

Penetrating cardiac injury will most commonly produce damage to the

A. right atrium

B. sino-atrial node

C. left ventricle

D. right coronary artery

E. right ventricle
E. right ventricle

what a stupid f-ing question. see wiki.
-----------
IC71 [?Aug12]

The first priority in managing a witnessed ventricular fibrillation cardiac arrest is

A. defibrillation times 3

B. endotracheal intubation

C. establishment of intravenous access

D. external cardiac massage

E. mask ventilation with 100% oxygen
D. external cardiac massage

see next slide.
-----------
THREE STACKED SHOCKS
Three Stacked Shocks were removed from the standard ALS algorithm as part of the 2010 CoSTR changes. It is now included under the special circumstances in resuscitation guideline.
Where a patient with a perfusing rhythm develops a shockable rhythm in a witnessed and monitored setting and the defibrillator is immediately available and they were previously well perfused and oxygentated pre-arrest then the use of 3 stacked shocks may be considered.
This situation is rare and may occur in the pre-hospital setting, emergency departments, critical care and coronary care units, and possibly also in the operating room. In these settings it may be appropriate to use a 3 stacked-shock technique, especially where there may be a relative contraindication to external cardiac compressions (e.g. after cardiac surgery).
The “3 stacked-shock sequence” can be optimized by immediate rhythm analysis and charging of the defibrillator. This sequence may be of benefit in scenarios where the time required for rhythm recognition and for recharging the defibrillator is short (ie:. <10 seconds). In these situations, such as in-hospital arrests, it would be expected to deliver the sequence of shocks (up to three) in no more than 30 seconds.
IC87 ANZCA Version [Apr07][Jul07][Mar10][Aug12?]

A 30-year-old man presents to the emergency department following a high speed motor vehicle accident. His blood pressure is 70/50 mmHg with a strongly positive FAST (focused abdominal sonography for trauma). His chest X-ray shows a widened mediastinum. The most appropriate method to assess the widened mediastinum in this patient is

A. aortogram

B. CT angiogram of the chest

C. repeat chest X-ray

D. intraoperative TOE (transoesophageal echocardiography)

E. transthoracic echocardiography
D. Intraoperative TOE (transoesophageal echo)

see wiki.
Note: FAST incorrectly defined - A = assessment NOT abdominal. FAST looks at 4 areas of abdo and pericardium (Left flank-spleen, right flank-liver, suprapubic-bladder and subxiphoid-pericardial fluid/tamponade).
Pt is unstable and needs laparotomy.
-----------
IC90 ANZCA version [Jul07][Sep11][?Aug12]

The best indicator of adequate fluid resuscitation in the trauma patient is:

A arterial pH

B blood pressure

C core temperature

D pulse rate

E serum lactate level
E serum lactate level
--
Serum lactate determination is a reliable marker of hypoperfusion in hemorrhagic shock. Lactate levels are a measure of anaerobic metabolism secondary to inadequate DO 2 . The amount of lactate produced is believed to correlate with the total oxygen debt, the magnitude of hypoperfusion and thus, the severity of shock. Although initial lactate levels may not correlate with outcome, the ability to clear lactate to normal seems to predict adequate resuscitation. Serum lactate levels also can be followed as the resuscitation continues. The normal plasma lactate concentration is 0.5 to 1.5 mmol/L; levels above 5 mmol/L indicate significant lactic acidosis. The half-life of lactate is approximately 3h; thus the level decreases rather gradually after correction of the cause. Failure to clear lactate within 24 hours after circulatory shock is a predictor of increased mortality.

Fluid resuscitation in trauma
Year : 2006 | Volume : 10 | Issue : 4 | Page : 241-249
-----------
MC38 [Aug12]

Causes of inverted P waves in Lead II of the electrocardiogram include

A. transposed lower limb leads

B. junctional rhythm

C. hypothermia

D. left axis deviation

E. inferior myocardial infarction
B. junctional rhythm

– retrograde activation of atria, originating from the a-v node
- may also have smaller PR interval

~~

C. hypothermia – Osborn waves - a rounded hump at the J point
D. left axis deviation – positive in 1, negative in aVF
-----------
MC42 ANZCA version [apr04][Mar11][Sep11][Aug12]

Abnormal Q waves are NOT a feature of the ECG in

A. an old myocardial infarction

B. left bundle branch block

C. recent transmural myocardial infarction

D. digitalis toxicity

E. Wolff-Parkinson-White syndrome
D. digitalis toxicity

Digoxin is associated with ST depression and other arrhythmias and calcium channel blockers with AV conduction disturbances. (harrison’s 1267-70) Q waves: transmural MI, LBBB, hyperkalaemia, WPW, amyloid, HOCM, cardiac contusion, myocarditis, dextrocardia, reverse limb leads. Epstein page 52.
-----------
MC59 [Aug12]

The treatment LEAST likely to be useful for torsades de pointes is

A. defibrillation

B. procainamide

C. magnesium

D. electrical pacing

E. isoprenaline
B. procainamide

~~
Avoid Class I anti-arrhythmics, mexilitine & sotalol (in renal failure)

Magnesium – first line treatment according to emedicine
Isoprenaline – second line treatment according to emedicine
Lignocaine – has been used to treat resistant TDP
Phenytoin – useful, as it shortens the QTc – according to Harrisons

β blockade is the mainstay for congenital prolonged QT (Lown Ganong Levine)
– remember that fat Med Reg in Dunedin on his first on call night in Nelson



-----------
M6,1404
Torsades de pointes, which may mimic VF or VT, is a life threatening arrhythmia that occurs in the presence of disturbed repolarisation (hence its association with prolonged QT interval). Discontinuation of drugs that predispose to QT interval prolongation and correction of electrolyte abnormalities are essential in the treatment of torsades de pointes. Acute therapy may include defibrillation, 1 to 2g of intravenous magnesium sulfate, amiodarone (n.b not considered a good therapy in most texts ), isoprenaline and overdrive pacing.

By mechanisms that are poorly understood, mexiletine
may exacerbate existing arrhythmias and may evoke torsades de
pointes.

AMIODARONE - prolongation of the QT interval is commonly
observed during chronic treatment but induction of torsades de
pointes is seen only rarely.

The most important unwanted effect of sotalol may be its
arrhythmogenic activity, which includes provoking torsades
de pointes. The likelihood of this effect is increased by hypokalaemia,
and is greater in women than in men. Since sotalol
is eliminated mainly by excretion, unchanged in the urine, the
danger of torsades de pointes is also increased where there is
renal dysfunction.




http://www.lhsc.on.ca/critcare/icu/cctc/procprot/pharmacy/mono2/procainamide.html
Adverse Effects of procainamide
Cardiac:
o arrhythmias: ventricular tachycardia or fibrillation, Torsades de Pointes (associated with high levels of procainamide and/or NAPA (N-acetyl procainamide); secondary to prolonged QT interval)
o heart block
o prolongation of PR, QRS, QT intervals (up to 50%)
o bradycardia, asystole
o decreased BP, decreased
MC77 [Jul97] [Aug10][Sep11][?Aug12]

Which of the following does not cause diastolic dysfunction (type A)

A. Hypertension

B. Ischaemic heart disease

C. Aortic stenosis

D. Adrenaline

E. Myocardial fibrosis

(see also MC108)
D. Adrenaline


The major causes of diastolic dysfunction include:
• chronic hypertension
• hypertrophic cardiomyopathy
• aortic stenosis
• coronary artery disease
• restrictive cardiomyopathy (a rare condition in which the heart muscle is infiltrated, and made stiff, by abnormal cells, protein, or scar tissue. The most common cause of restrictive cardiomyopathy is amyloidosis, a disease in which protein-like substance is deposited within the body's tissues. Other causes include sarcoidosis and hemochromatosis.)
• aging (Whether age alone causes stiffening of the ventricles, or whether such stiffening is related to "subclinical hypertension" or some other definable medical condition, is not yet worked out.)
-----------
ME23 [May09][Oct09][Aug10][?Aug12]

Compared to blood glucose concentration, plasma glucose concentration is approximately

A. 32% lower

B. 14% lower

C. the same

D. 14% higher

E. 32% higher
D. 14% higher than blood glucose concentration
-
Just remember that RBCs use up glucose!


-----------
ME47 [Mar12][Aug12]

Which of the following are feature of Conn’s syndrome?

A. Normoglycaemia, hypernatremia , hypokalemia
B. Hypoglycaemia, hypernatremia, hypokalemia
C. Hyperglycaemia, hyponatremia, hyperkalemia
D. Normoglycaemia, hyponatremia, hyperkalemia
E. Hypoglycaemia, hyponatremia, hyperkalemia
A. Normoglycaemia, hypernatremia , hypokalemia
--
see wiki (tendency for impaired glucose tolerance…)
Also tend to have low magnesium.
-----------
MG05 [Jul07][Mar10][Aug12]

All of the following may be associated with ulcerative colitis EXCEPT

A. cirrhosis

B. iritis

C. psoriasis

D. arthritis

E. sclerosing cholangitis
C. psoriasis


get psoriasis with crohns but not UC
-----------
MH32 [Mar12][Aug12]

Iron deficiency is characterised by

A. high serum ferritin level and low serum iron

B. high serum ferritin level and absent bone marrow iron

C. increased serum ferritin level and normal serum iron

D. low serum ferritin level and low serum iron

E. low serum iron level and lowered total iron binding capacity
D. low serum ferritin level and low serum iron


-----------

D. Absence of Fe in bone marrow – probably a more definitive test for iron deficiency.

D also tru-ish - if it said "stainable iron"

SD472
In adults, iron deficiency is usually due to chronic loss leading to depletion of iron stores. Parturients are susceptible because of increased RBC mass during gestation and the needs of the foetus for iron. Symptoms depend on the actual haemoglobin concentration.
Patients experiencing chronic loss may not be able to absorb sufficient iron from the GIT to form Hb as rapidly as RBCs are lost. As a result, RBCs are often produced with too little Hb, resulting in microcytic hypochromic anaemia. The absence of stainable iron in the bone marrow aspirates is confirmatory for iron deficiency, decreased serum ferritin concentrations serve as a cost effective alternative to bone marrow examinations for diagnosis.
MH60 ANZCA version [Apr08][Oct08][Sep11][Mar12][?Aug12]

Haemophilia A is commonly associated with:

A. a haemarthrosis in a female infant

B. a haemarthrosis in a male infant

C. low levels of Factor IX

D. normal prothrombin time (PT) and prolonged activated partial thromboplastin time (APTT)

E. prolonged prothrombin time (PT) and prolonged activated partial thromboplastin time (APTT)
D. normal prothrombin time (PT) and prolonged activated partial thromboplastin time (APTT)
--
Factor VIII deficiency, 1 in 10,000 males, X linked recessive, females mostly carriers, increased APTT, normal PT

A - True for severe haemophilia A, but not exclusively female (X linked, so unlucky to have as a female)
B - True for severe haemophilia A, but not exclusively male
C - Congenital factor VIII deficiency
D - True (as extrinsic pathway is unaffected, PT is normal)
E - Normal PT
-----------
MZ69 ANZCA version [2004-Apr] Q95, [2004-Aug] Q10 [Mar12][Aug12]

A 32 year-old man has a four day history of progressive weakness in the extremities. He has been well apart from an upper respiratory tract infection 10 days ago. His temperature is 37.8oC and respiratory rate 42 breaths.min-1 and shallow. He has symmetric weakness on both sides of his face and the proximal and distal muscles of the extremities. Sensation is intact. His most likely diagnosis is

A. acute disseminated encephalomyelitis

B. Guillain-Barre syndrome

C. myasthenia gravis

D. poliomyelitis

E. polymyositis
B. Guillain-Barre syndrome


Diagnostic criteria:

Required
• Progressive weakness of 2 or more limbs due to neuropathy
• Areflexia
• Disease course < 4 weeks
• Exclusion of other causes (see below)

Supportive
• relatively symmetric weakness
• mild sensory involvement
• facial nerve or other cranial nerve involvement
• absence of fever
• typical CSF findings
• electrophysiologic evidence of demyelination
-----------
NA15 ANZCA version [Mar92] [Aug92] [Mar93] [Aug93] [2002-Mar] Q18, [2002-Aug] [Mar10] [Aug10] [Aug12]

The skin of the anterolateral part of the gluteal region, between the iliac crest and the greater trochanter, is supplied by the

A. Ilioinguinal nerve

B. Genitofemoral nerve

C. Superior gluteal nerve

D. Subcostal nerve

E. Lateral cutaneous nerve of the thigh

F. Femoral nerve
D. Subcostal nerve


Subcostal nerve = T12 intercostal = MUFFIN TOPS (see picture below)

Each nerve from T7 to T12 also gives off a lateral cutaneous branch (with anterior and posterior branches), which divides in the mid-axillary line. These branches supply the skin of the flank and back in the relevant distribution. The iliohypogastric and subcostal nerves, however, do not have a divided lateral cutaneous nerve, but continue down to supply the skin over the upper lateral buttock. The ilioinguinal nerve has no lateral cutaneous branch.

Concise Anatomy for Anaesthesia


"supplies sensory innervation to the skin over the hip."
Wikipedia
-
A. ilioinguinal nerve - false
B. genito-femoral nerve - false
C. superior gluteal nerve - false: "The superior gluteal nerve (L4, 5, S1) accompanies the superior gluteal vessels as the only structures that pass through the upper compartment of the greater sciatic foramen (above piriformis). It supplies gluteus medius and minimus and tensor fasciae lata." (Ellis)
D. subcostal nerve - true: "The 12th thoracic (subcostal) nerve runs along the lower border of the 12th rib below the subcostal vessels, passes behind the lateral arcuate ligament to run in front of quadratus lumborum behind the kidney and colon. The nerve then passes between transversus abdominis and internal oblique and then has a course and distribution which are similar to the lower intercostal nerves. However, there is one point of difference: the lateral cutaneous branch of the 12th nerve descends without branching to supply the skin over the lateral aspect of the buttock" (Ellis)
E. lateral cutaneous nerve of thigh - false: "The anterior branch supplies the skin over the antero-lateral aspect of the thigh down to the knee, where it links up with twigs from the intermediate cutaneous nerve of the thigh and the infrapatellar branch of the saphenous nerve to form the patellar plexus. The posterior branch penetrates the fascia lata to innervate the skin of the lateral aspect of the leg from the greater trochanter to the mid-thigh."




-----------
New: Appropriate postoperative maintenance fluid in a child [can't recall situation, but something to do with head injury]:

A. 3% and 1/3 NS
B. 1/2 NS
C. Normal Saline
D. Hartmanns
E. Hartmanns with glucose
C. Normal Saline
-----------
New: At what valve area do you begin to get symptoms, at rest, with mitral stenosis?

A. 1.5 cm2
A. 1.5 cm2
--
From "Modern Management of Mitral Stenosis" Circulation July 2005
Table 1
Moderate MS with MVA 1.2-1.6 cm2; Gradient 4-9 mmHg; Normal PAP; Class II symptoms.
-----------
New: Buprenorphine patch taken off. Time taken for plasma concentration to halve?
12 hours
-
From Product Information Norspan patch. "After removal of a nor span patch, buprenorphine concentrations decline, decreasing approximately 50% in 12 hours (range 10-24h).
-----------
New: Cerebral oximetry measures?

A. Arterial saturation
B. Mostly arterial saturation and some venous saturation
C. Capillary saturation
D. Mostly venous saturation and some arterial saturation
E. Venous saturation
D. Mostly venous saturation and some arterial saturation
-
ScO2 is weighted for ~70-75% venous blood.
-----------
New: Day 4 epidural. On 40 mg SC enoxeparin daily postoperatively (8 pm). When is the most appropriate time to remove the epidural?

A. Day 5 at 12 midday
B. Day 5 at 6 am
C. Day 5 at 6 pm
D. Day 6 at ?
A. Day 5 at 12 midday
--
From APMSE 3rd Ed 2010 pg197:
'Epidural catheter should be removed at least 10-12 hours after the last dose of LMWH and the next dose should not be given until at least 2 hours after removal. Concurrent administration of other drugs that may affect haemostasis (e.g. anti platelets) should be avoided. Epidural catheters should be removed at least 2 hours before TWICE DAILY LMWH dose regimens are started.'
-----------
New: Incidence of fat embolism following closed femoral fracture?
Patients with a single long bone fracture have a 1-3% chance of developing the syndrome, but it has been reported in up to 33% of patients with bilateral femoral fractures. CEACCP 2007 article
-----------
New: Less blood wastage if:

A. Lower transfusion threshold
--
Do they mean blood wastage in terms of blood bank or inappropriate transfusion?

From Periop Pt blood mx guidelines 2012:
Practice point (PP) 2 - In the absence of acute myocardial or cerebrovascular ischaemia, postop transfusion may be inappropriate pts with Hb>80g/L
PP3 - Patients should not receive a transfusion when the Hb level is > 100g/L. In postop pts with acute myocardial or cerebrovascular ischaemia and a Hb 70-100g/L, transfusion of a single unit of RBC followed by reassessment of clinical efficacy is appropriate.

From AABB - recommendation 2: suggest restrictive strategy in hospitalised pts with preexisting CVS disease and considering transfusion for pts with symptoms or a Hb of 8g/dL or less (weak recommendation)
-----------
New: Loading dose of IV paracetamol in x kg child?

A. [dose corresponding to 20 mg/kg]
-----------
New: Maximum dose of local infiltration of 0.5% bupivacaine in an x kg child?

A. [dose corresponding to 2.5 mg/kg; there was no option corresponding to 2 mg/kg]
Maximum dose of plain solution is 2mg/kg and with epinephrine is 3mg/mg
-----------
New: Meconium stained liquour but neonate delivered is vigorous. Rationale for NOT suctioning the neonate?

A. May aspirate meconium
B. May cause bradycardia
C. May cause hypertension
B. May cause bradycardia
-
see uptodate article re: meconium stained amniotic fluid.
-----------
New: New onset AF. For what period of time is it safe to perform DCCV without prior TOE to exclude thrombus?

A. <24 hours
B. <48 hours
B. <48 hours
-----------
New: Normal systolic BP at birth?

A. Something less than 70 mmHg
B. 70 mmHg
C. 85 mmHg
D. Something more than 85 mmHg
E. 115 mmHg
c. 85mmHg
-
80/50 is normal

-----------
New: Off-label use of a drug refers to all of the following EXCEPT:

A. Different age-group
B. Different indication
C. Different concentration
D. Different route of administration
C. Different concentration

-----------
New: Perform a brachial plexus block however the medial forearm is NOT numb. Which nerve has been missed?

A. Medial brachial cutanous nerve
Medial ANTEbrachial cutaneous nerve (C8,T1) supplies the medial forearm.
-
C8,T1 join to form the LOWER TRUNK -> divides to anterior and posterior division -> ANTERIOR division forms the MEDIAL CORD. From the Medial cord -> medial pectoral nerve, medial cutaneous nerve of arm (or medial brachial cutaneous nerve) and MEDIAL ANTEBRACHIAL CUTANEOUS NERVE (or medial cutaneous nerve of the forearm.

-----------
New: Performing a caudal block in a child and add clonidine to prolong duration of block. What significant complication is increased?

A. Sedation
B. Urinary retention
A. Sedation
-----------
New: Performing a caudal block in a child/neonate. What is the first sign of a total spinal anaesthetic?
?Apnoea - if under GA already
-----------
New: Photograph of an Arndt endobronchial blocker. Orifice labelled 'X'. What goes in 'X'?

A. Bronchoscope
-----------
New: Regarding mixed venous oxygen saturations:

A. 60% is normal
B. Can be used to calculate the CO
-----------
New: Regarding remifentanil. All of the following are true EXCEPT:

A. Metabolized by plasma cholinesterase
A. Metabolized by plasma cholinesterase
--
Metabolized by nonspecific plasma ESTERASES
-----------
New: Thoracodorsal nerve arises from?
Thoracodorsal (middle subscapular) nerve arises from the posterior cord and receives innervation from C6,7,8. It innervates the Latissimus dorsi muscle.

Roots->Trunks->Divisions->Cords->Branches
-----------
New: Which is a specific PDE inhibitor?

A. Theophylline
B. Dipyridimole
C. Milrinone
C. Milrinone
-
Theophylline is a nonselective PDE inhibitor

Dipyridamole inhibits adenosine deaminase and phosphodiesterase, preventing degradation of cAMP an inhibitor of platelet function. It also directly stimulates the release of prostacyclin.

Milrinone is a PDE3 inhibitor
-----------
New: Which of the following does NOT occur following bilateral lung transplant?

A. Impaired mucociliary clearance
B. Impaired lymphatic drainage
C. Impaired HPV
C. Impaired HPV
-
From http://www.ispub.com/journal/the-internet-journal-of-anesthesiology/volume-20-number-1/anesthetic-challenges-in-patients-after-lung-transplantation.html#sthash.Rkdzssv1.dpbs
A. false - mucociliary clearance is impaired in pulmonary allograft (together with immunosuppression and impaired cough, places the patient at an increased risk for preoperative pneumonia
B. false - Interruption of lymphatic drainage and leaky pulmonary capillaries increases the susceptibility to pulmonary edema.
C. TRUE - Hypoxic pulmonary vasoconstriction is intact in the pulmonary allograft so during acute rejection pulmonary bloodflow may be directed away from the transplanted lung. In patients with SLT - 60-70% of pulmonary perfusion is directed towards the transplanted lung.
-----------
New: White cylinder with grey shoulder?

A. CO2
B. Air
C. O2
D. N2O
E. N2
A. CO2
--
B. air - false - white body, black and white shoulder
C. O2 - false - white body
D. N2O - false - blue shoulder, white body
E. N2 no idea.

ref: Chris Thompson Notes - The Anaesthetic machine - gas supplies
-----------
New: With regard to Digoxin toxicity which of the following is NOT a feature?

A. Atrial flutter
Mobitz type II block (if it is an option!)
--
From Crit Care Med 2010 Vol. 38 No. 6 (Suppl.) Drug induced arrhythmias:
"Digoxin toxicity can precipitate almost any type of arrhythmia, some are more common than others. Due to its underlying mechanism of action, ectopy in the form of ventricular premature depolarizations is relatively common, though non-specific. Similarly, conduction block of any degree (ALTHOUGH RARELY MOBITZ TYPE II) can also occur."

Arrhythmias that have been considered pathognomic for digoxin toxicity include paroxysmal atrial tachycardia with block, accelerated junctional rhythm and bidirectional VT.

See also Digitalis Toxicity: a fading but crucial complication to recognise - Am J Med 2012 Table.
-----------
NN05 ANZCA version [2004-Aug] Q17, [2005-Apr] Q63, [Apr07] [Sep11][?Aug12]

The carotid sinus derives its nerve supply from the

A. vagus nerve

B. glossopharyngeal nerve

C. ansa cervicalis (hypoglossi)

D. middle cervical ganglion

E. stellate ganglion
B. glossopharyngeal nerve


● Carotid nerve (branch of glossopharyngeal) – runs down the internal carotid artery and supplies the carotid sinus (pressor) and body (chemo)
Vagus nerve is efferent.

-----------
NN14 ANZCA version [Mar93] [Aug93] [Aug96] [Jul98] [Apr99] [2004-Aug] Q28, [2005-Apr] Q34, [Jul05] [Jul07] [?Aug12]

The muscles of the larynx which separate the vocal cords are the

A. thyro-arytenoids

B. lateral crico-arytenoids

C. oblique arytenoids

D. posterior crico-arytenoids
D. posterior crico-arytenoids
-
"The posterior cricoarytenoid muscle arises from the posterior surface of the lamina of the cricoid and is inserted into the posterior aspect of the muscular process of the arytenoid. It abducts the cord by external rotation of the arytenoid and thus opens the glottis; it is the only muscle to do so." (Ellis, Anatomy for Anaesthetists, 8ED, p34)

● Posterior crico-arytenoid – opens the glottis by the abducting cords
● Lateral crico-arytenoid – closes the glottis by the adducting cords

Concise Anatomy for Anaesthesia
-----------
NV42 [Apr07] [Jul07] [Oct08][Mar10] [Aug10] [Sep11] [Mar12][Aug12]

C6/7 motor function causes (or myotome of C6-7):

A. Flexion/extension of fingers

B. Flexion/extension of wrist

C. Flexion of elbow

D. Elbow pronation/supination

E. Shoulder external rotation/abduction
B. Flexion/extension of wrist



Shoulder
- ABduction C5 6
- ADDuction C7 8
Elbow
- Flexion C5 6
- EXTension C7 8
Wrist
- Flex C6 7
- EXT C7 8
Fingers
- Flex C7 8
- EXT C7 8
- Abduction / Adduction T1

-----------
PC54 [Oct09][Aug12?]

Which drug is an example of a specific PDE3 Inhibitor?

A. Aminophylline

B. Sildenafil

C. Milrinone

D. Dipyridamole
C. Milrinone


Sildenafil is a PDE5I
-----------
PI66 [Mar12] [Aug12]

The correct ranking of fat/blood partition co-efficients, in order of increasing solubility in fat, for sevoflurane (S), isoflurane (I), desflurane (D) and nitrous oxide (N2O) is

A. N2O < D < S < I

B. D < N2O < S < I

C. D < N2O < I < S

D. N2O < D < I < S

E. N2O < S < D < I
D. N2O < D < I < S


Fat:Blood Partition Co-efficients:
---------------------------
N2O < Des < Iso < Sevo
2.3 27 45 48


Muscle:Blood Partition Co-efficients:
------------------------------
N20 < Des < Iso < Sevo
1.2 2.0 3.1 4.0
-----------
PI83 [Mar10][Aug12]

Desflurane vaporiser, heated because of

A. High SVP

B. High boiling point

C. Low SVP

D. High MAC

E. Low MAC
A. High SVP

see wiki.
Des also has a low BP not high BP.
-----------
PP ANZCA version [Apr08][Oct09][Aug10][Aug12]

A 12-year-old boy presents at 11 pm having fallen and dislocated his hip at 4 pm. One hour after the injury he ate a meal. He will require closed reduction of the dislocation. The BEST anaesthesia option is to:

A. defer the case until the next morning and keep him fasted

B. perform a femoral nerve block

C. perform an inhalational induction and then maintain spontaneous ventilation using a face mask

D. perform a rapid sequence induction with cricoid pressure and intubate the trachea

E. provide intravenous sedation
D. perform a rapid sequence induction with cricoid pressure and intubate the trachea
-

Procedure needs to be done urgently, child ate after injury = NON-fasted

"Dislocation of the hip requires immediate reduction to avoid injury to the femoral head and to relieve patient discomfort."

Textbook of Pediatric Emergency Procedures
Christopher King 2007, p978
-----------
PP66 ANZCA version [2003-Apr] Q108, [2003-Aug] Q56, [Mar06] [Jul06] [Mar10] [Aug12]

In infants with congenital pyloric stenosis

A. dehydration is associated with early hyponatremia

B. plasma chloride levels seldom fall below 85 mmol.1-I

C. renal conservation of hydrogen and potassium ions occurs

D. the urine is initially alkaline, then may become acidic

E. vomiting causes a loss of potassium ions
D. the urine is initially alkaline, then may become acidic


Initially: vomit acid, compensate by alkaline urine. As volume becomes contracted, retain Na+ and HCO3 and lose K and H. Develop a ketoacidosis from starvation and lactic acidosis from shock.
-----------
PP92 [Apr07][Apr08][Aug12]

5yo 35kg child having repair of leg laceration. Gas induction with sevoflurane, N2O and oxygen. Can't get in drip. Put in LMA and immediately get stridor and airway obstruction and desaturate to 90%. Next step after increase FiO2 to 100% is:

A. Remove LMA and deepen with sevoflurane

B. Leave LMA and deepen with sevoflurane

C. Intralingual Suxamethonium

D. IM Atropine

E. IM Suxamethonium
A. Remove LMA and deepen with sevoflurane
-----------
PZ98 [Mar12][Aug12]

Cephalothin has NO significant activity against

A. escherichia coli

B . proteus mirabilis

C. pseudomonas aeruginosa

D. staphylococcus aureus

E. streptococcus pneumoniae
C. pseudomonas aeruginosa


MIMS:

The in vitro bactericidal action of cephalothin results from inhibition of cell wall synthesis. In general cephalothin has higher activity against Gram positive than Gram negative organisms. Gram negative organisms vary greatly in their sensitivity to the drug. Susceptibility testing is highly desirable because the range of antibiotic concentrations at which bacteria are inhibited may vary substantially according to the isolate.

Cephalothin is usually active against the following organisms in vitro: beta-haemolytic and other Streptococci (most strains of Enterococci, e.g. Enterococcus faecalis, are resistant); Staphylococci (including coagulase positive, coagulase negative, and penicillinase producing strains; methicillin resistant Staphylococci are resistant); Streptococcus pneumoniae; Haemophilus influenzae; Escherichia coli; Klebsiella; Proteus mirabilis.


-----------
PZ118 [Jul05] [Mar06] [Apr07] Q80, [Jul07] [?Aug12]

In patients with renal failure each of the following drugs has prolonged clearance or has active metabolites with prolonged clearance EXCEPT

A. aspirin

B. buprenorphine

C. codeine

D. pethidine

E. tramadol
B. buprenorphine


Of the opioids, buprenorphine poses the least risk to patients with renal failure as the metabolites are virtually inactive and if accumulation does occur it is of no significance.

Update in Anaesthesia
Issue 7 (1997) Article 2: Page 5 of 7
The Management of Postoperative Pain


"Morphine and codeine are avoided in renal failure/ dialysis pts; Hydromorphone and oxycodone are used with caution and close monitoring; and that methadone and fentanyl/sufentanil appear to be safe to use".
also stated "insufficient data to make recommendation" on use of oxycodone in renal failure, also stated "use of oxycodone best avoided in dialysis pts".

Opioids in Renal Failure and Dialysis Pts
Journal of Pain and Sx Mx" Vol 28. No5, Nov 2004.

-----------
Rpt: A Full Size C oxygen cylinder has pressure downregulated from?

A. 16,000 kPa to 400 kPa
B. 16,000 kPa to 240 kPa
C. 11,000 kPa to 400 kPa
D. 11,000 kPa to 240 kPa
A. 16000 kPa to 400 kPa
-
See RAH presentation.
-----------
Rpt: According to ANZCA-endorsed labelling standards a brachial plexus catheter should be labelled?

A. Yellow
-----------
Rpt: Best single predictor of difficult intubation in obese patient?

A. Mallampati score
B. Interincisor distance
C. Severe OSA
D. increased soft tissue in suprathyroid region (?Option in Aug12)

see also AZ03 - different choices
D.?
--
c. false -
-----------
Rpt: Dural Sac ends at what level in a neonate?

A. L1
B. L3
C. L5
D. S1
E. S3
E. S3
-
-----------
Rpt: Intraoperative pediatric arrest during scoliosis surgery most likely due to?

A. Underappreciated degree of blood loss
?A. Underappreciated degree of blood loss
-
difficult to find references. From Anaesth Intensive Care 2010;38:1088-1084:
'Fourteen cardiac arrests were recalled and details were specified for four: hyperkalaemia, major haemorrhage and CVS collapse in one patient with cerebral palsy and three DMD patients. Ten deaths were recalled, detail specified for six: three associated with major haemorrhage or transfusion (one DMD in the arrest group and one DMD with TRALI, one idiopathic scoliosis patient), one AMI in a pt with cardiomyopathy but no other myopathy, one brainstem stroke (with thoracic spin bifid a) and one due to sepsis. Closed questions on venous/arterial thrombosis, infection (wound or instrumentation), failed fusion rates, awareness or intraoperative recall during wake-up tests were not included.'

-----------
Rpt: Most common cause of awareness?
(see AC62)

A. Failure to check apparatus
From Anesthesia and Analgesia Feb 2009 Vol. 108(2) p527-35:
'This review suggested two risk factors for awareness during anaesthesia. The first is the use of light anaesthesia. The second is the presence of a history of awareness. ..
Difficult and prolonged layngoscopy and intubation were associated with awareness in only 4.5% of the cases. Considering that the overall incidence of difficult intubation varies from 4.5-7.5%, it was not found to play a major role as a risk factor. Administration of supplemental doses of induction hypnotics should prevent the patients from regaining consciousness when this problem arises. ..
The major cause of awareness was overly light anaesthesia at the time of the episode. Anesthetic machine malfunction or misuse and increased anaesthetic requirement were much less frequent.'
-----------
Rpt: Risk factors for postoperative ulnar nerve injury?

A. Contralateral arm
From Miller's Anaesthesia:
'The aetiology of ulnar nerve palsy is multifactorial and not always preventable. In a large retrospective review of preoperative ulnar neuropathy lasting longer than 3 months, the onset of symptoms occurred more than 24 hours postop in 57% of patients, 70% were male, and 9% had bilateral symptoms. Very thin or obese patients were at increased risk, as were patients on prolonged postop bed rest. No association with intraop patient position or anaesthetic technique. ..
Other risk factors, including diabetes mellitus, vitamin deficiency, alcoholism, cigarette smoking and cancer need further studies to be substantiated.'
-----------
Rpt: Urgent reversal of INR 4.5. Intern already gave vitamin K.

A. FFP
B. Prothrombinex
C. Prothrombinex AND FFP
C. Prothrombinex and FFP
-

-----------
Rpt: When do most patients with SAH rebleed?

A. 0-24 hours
A. 0-24 hours
--
According to emedicine: 'Rebleeding is the most dreaded early complication of SAH. The greatest risk of rebreeding occurs within the first 24 hours of rupture (4.1%). The cumulative risk of rebreeding is 19% at 14 days. The overall mortality rate from rebreeding is reported to be as high as 78%. '
-----------
RU12 ANZCA version [1986] [Aug93] [Mar94] [Apr96] [2002-Aug][2003-Apr][2005-Apr][Jul05][Mar12][Aug12]

If a patient experiences parasthesia in the little finger during supraclavicular brachial plexus block, the needle is in proximity to the

A. posterior cord

B. middle trunk

C. ulnar nerve

D. lower trunk

E. medial cord
D. lower trunk


*Anatomy of this area is such that you block trunks at a supraclavicular level

C. Ulnar nerve – same effect, but doesn’t form until well below the clavicle (see EF157)
E. Medial cord – formed at the apex of the axilla, well below the clavicle

~~
-----------
* Anatomy of this area is such that you block trunks at a supraclavicular level.
* NYSORA site says that supraclavicular targets the trunks. Divisions are behind the clavicle and cords are infraclavicular. OHA (1st Ed) p 1014 implies supraclavicular is at the root/trunk level.--Phil 03:40, 25 Jul 2007 (EDT)

http://www.backpain-guide.com/Chapter_Fig_folders/Ch05_Anatomy_Folder/Ch5_Images/05-9_Brachial_Plexus.jpg
SC13 [Oct08][May09][Aug12]

The most appropriate investigation to diagnose Type A aortic dissections in potentially unstable patients is

A. angiography

B. CAT scan

C. magnetic resonance imaging (MRI)

D. transoesophageal echocardiography

E. transthoracic echocardiography
D. transoesophageal echocardiography
--
Much debated on wiki!

Essentials of Cardiac Anaesthesia by Kaplan on pg 412
"Acute aortic dissection is a medical emergency, and the medical management and diagnostic evaluation of a patient with suspected aortic dissection should proceed simultaneously. Time is of the essence because an initial mortality of 3% when surgery is expedited increases to as high as 20% when preoperative preparation is prolonged and diagnostic testing delays the start of the surgery. Patients who are unstable with a high likelihood of aortic dissection based on clinical evaluation and existing diagnostic studies should be transported immediately to the operating room where TOE can be performed to verify the diagnosis and surgery can proceed immediately thereafter."
-
Two classifications in common use:
Stanford - Type A and B depending on whether ascending or descending aorta involved
DeBakey - Divided into Types I, II and III as above
-----------
SF53 ANZCA version [2001-Apr] Q6, [2001-Aug] Q4, [2003-Aug] Q66, [2004-Apr][Apr08][Oct09][Mar 10][Aug10][Sep11][?Aug12]

Carbon dioxide is the most common gas used for insufflation for laparoscopy because it

A. is cheap and readily available

B. is slow to be absorbed from the peritoneum and thus safer

C. is not as dangerous as some other gases if inadvertently given intravenously

D. provides the best surgical conditions for vision and diathermy

E. will not produce any problems with gas emboli as it dissolves rapidly in blood
C. is not as dangerous as some other gases if inadvertently given intravenously


"Carbon dioxide is used as the insufflation gas as it is nonflammable, colourless and has a higher blood solubility than air, thus reducing the risk of complications after venous embolism."

Continuing Education in Anaesthesia, Critical Care & Pain | Volume 4 Number 4 2004
-----------
Y&A p865
CO2 is the insufflating gas of choice because it is non-flammable, does not support combustion, readily diffuses across membranes, is rapidly removed in the lungs and is highly soluble because of rapid buffering in blood. The risk of embolization is small. As much as 200ml of CO2 injected directly into a peripheral vein may not be lethal, whereas only 20 ml of air may prove to be so. In addition, CO2levels in blood and expired air can be easily measured, and its elimination can be augmented by increasing ventilation. As long as oxygen requirements are met, a high concentration of blood CO2 can be tolerated. Also, medical grade CO2is readily available and inexpensive.

The disadvantages mainly stem from the fact that CO2 is not inert. It causes direct peritoneal irritation and pain during laparoscopy under LA because it forms carbonic acid when in contact with the moist peritoneum. In addition, CO2 is not very soluble in the absence of RCs and therefore can remain in gaseous form intraperitoneally after laparoscopy causing referred shoulder pain. Hypercarbia and respiratory acidosis occur when the buffering capacity of blood is temporarily exceeded. In addition, CO2 exerts widespread local and often contradictory systemic effects that may manifest overall as hypertension, tachycardia, cerebral vasodlation, increased CO, hypercarbia and respiratory acidosis.
SF87 [Mar10] [Aug10] [?Aug12]

Labour epidurals increase maternal and foetal temperature. This results in neonatal

A. Increased sepsis

B. Increased investigations for sepsis

C. increased non shivering thermogenesis

D. Increased need for resuscitation

E. Cerebral palsy
B. Increased investigations for sepsis

Labour epidural analgesia is associated with an increase in maternal core body temperature, but also with an increased neonatal temperature and fetal heart rate. Several studies have shown that labour epidural analgesia is associated with increased neonatal neonatal sepsis evaluations, but no increase in neonatal sepsis.

From Wiki (quoting Chestnut's Obstetric Anesthesia 4th ed pg 457)
-----------
SG30 ANZCA version [1985] [1987] [Mar93] [Aug96] [Apr97] [Jul00] [2001-Apr][Mar12][Aug12]

A patient has suffered flash burns of the upper half of the left upper limb, all of the left lower limb and the anterior surface of the abdomen. The approximate percentage of the body surface which has been burned is:

A. 18%

B. 23%

C. 32%

D. 41%

E. 48%
C. 32%


Upper half of the left upper limb = 0.5 × 9 = 4.5%
All of the left lower limb = 18%
Anterior surface of the abdomen = 0.5 × 18 = 9%
TOTAL ≈ 32%
-----------
SG56 [Jul05][Mar10] [??Aug12]

During surgery there is increased secretion of each of the following hormones EXCEPT

A. aldosterone

B. glucagon

C. growth hormone

D. thyroid stimulating hormone

E. vasopressin
D. thyroid stimulating hormone

Insulin and Thyroxine goes down
-----------
SG59 [Apr07][May09][Oct09][Mar10][Sep11][Aug12]

Blunt liver trauma can be treated non surgically if

A. No peritoneal signs

B. Low Grade injury on CT scan

C. Severe COPD

D. Haemodynamically stable

E. US confirms <500mls peritoneal fluid collection (i thought this was a paracentesis result)
D. Haemodynamically stable

It was concluded that non-operative management should be the initial approach to all patients with blunt liver injury if haemodynamic stability can be ensured. When continued bleeding can be safely ruled out, a period of intensive monitoring is warranted.

Non-operative management of blunt hepatic injury in multiply injured adult patients. Al-Mulhim A S. Mohammad H A H. Surg J R Coll Surg Edinb Irel 2003; 1: 81-85.
-----------
SG65 ANZCA version [Apr08][Mar12][Aug12]

During prolonged trendelenburg positioning there is:

a. No change in ICP
b. No change in IOP
c. Increased pulmonary compliance
d. Increased myocardial work
e. No increased pulmonary venous pressures
d. Increased myocardial work
--
a. false - increases ICP (Miller)
b. false - increases IOP (Miller)
c. false - decreases pulmonary compliance (Miller)
d. ?true - presumably increased preload and venous return results in increased cardiac output and therefore myocardial work.
e. ?false - presumably increased preload and venous return translates to increased pulmonary venous flow and pressure if resistance is unchanged
-----------
SN18 [Mar11][Aug12]

Absolute contraindication to sitting position for posterior fossa craniotomy for meningioma

A: Presence of patent ventriculo-atrial drain/shunt
B: PFO
C: Oesophageal stricture so transoesophageal echo placement is out
D: ?
E: ?
A: Presence of patent ventriculo-atrial drain/shunt
and
B: PFO if large!
--
From Anaesth Int Care 2005; 33:323-331
Table 3
Absolute contraindications to the sitting position
- PATENT VENTRICULO-ATRIAL SHUNT
- Severe CVS disease
- LARGE PFO or other pulmonary-systemic shunt
- Cerebral ischaemia when upright and awake
- Anaesthesia or surgical team unfamiliar with position
Relative
- small PFO
- extremes of age
- uncontrolled HT
- COAD (may not tolerate VAE as well if it occurs)
-----------
ST19 [Apr08][Oct08][Sep11][Mar12][Aug12]

A diagnostic test has a sensitivity of 90% and a specificity of 99% in detecting a certain disease. From this we can conclude that

A. the false positive rate of this test is 1%

B. the false negative rate of this test is 1%

C. the positive predictive value of this test is 90%

D. the negative predictive value of this test is 90%

E. this test would be a useful screening test for this disease
A. the false positive rate of this test is 1%


FPR = FP/TN+FP or 1 - Specificity

FNR = 1 - Sensitivity
-----------
ST22 [Sep11][Aug12]

Recognised weaknesses of systematic reviews include all of the following EXCEPT

A. publication bias

B. duplicate publication

C. study heterogeneity

D. inclusion of outdated studies

E. systematic review author bias
E. systematic review author bias



Meta-analysis has been criticized, and some of its potential weaknesses identified. These include publication bias (negative studies are less likely to be submitted, or accepted, for publication), duplicate publication (and therefore double-counting in the meta-analysis), heterogeneity (different interventions, different clinical circumstances) and inclusion of historical (outdated) studies.

Myles & Gin
p.114-115
-----------
TMP-100 [Mar10][Aug10][?Aug12] Compared to lignocaine, bupivacaine is

A. Twice as potent

B. Three times as potent

C. Four times as potent

D. Five times as potent

E. As potent
C. Four times as potent
-
If procaine =1, then lignocaine potency=2, and bupivacaine potency =8.

Therefore bupivacaine is 4 times as potent as lignocaine

-> ANSWER is C (Ref: Hemmings & Hopkins, 2nd ed, p394)

Other recommended texts (Primary exam pharm texts) give differing values compared to procaine, but all give a ratio of 4x potency for buipvacaine:lignocaine (Stoelting and Katzung)



-----------
TMP-101 [Mar10][Aug10][?Aug12] Aneurysm sugery. Propofol/remifentanil/NMDR. DOA monitoring (Entropy). MAP 70 , HR 70/min, State entropy 50, Response entropy 70. What do you do?

A. ?
B. Metaraminol
C. Check TOF
D. Nothing
E. Increase TCI.
C or E? (either NMB wearing off or anaesthesia lightens)
--
RE- fast acting including higher frequencies- to 47 Hz, which includes the frontalis muscle
SE- always less or equal to SE. estimates hypontic effect on brain

see wiki debate - some say E.
but for aneurysm surgery - you want them still ! ?Is TOF the most appropriate way to assess deep block - argument that it should be PTC.
-----------
TMP-113 [Mar10][Aug10][Aug12?]
The BEST agent to decrease gastric volume AND increase gastric pH before semi-urgent procedure
A. Omeprazole
B. Cimetidine
C. Ranitidine
D. Sodium citrate
E. Cisapride
C. Ranitidine
-
From wiki debate: According Stoelting's Co-exisiting disease - p281 rantitidine is the most potent agent for reducing volume and increasing pH within the time span, ie 1-1.5 hours. Omeprazole needs to be given the night before. There are other PPI than can be give immediately prior to surgery, but this was not an option. Therefore C in best answer.

Na citrate increases volume.
-----------
TMP-130 [Oct09][Aug12] Essential diagnostic criteria on ECG for LBBB

A. Loss of septal Q's in V5 and V6
B. RSR in V1
C. Large slurred S in V6
D. T-waves opposite to direction of QRS
E. QRS duration minimum 0.2 s
A. Loss of septal Q's in V5 and V6
-
Diagnostic Criteria
- QRS duration of 120 ms
- Dominant S wave in V1
- Broad monophasic R wave in lateral leads (I, aVL, V5-V6)
- Absence of Q waves in lateral leads (I, V5-V6; small Q waves are still allowed in aVL)
- Prolonged R wave peak time > 60ms in left precordial leads (V5-6)


-----------
TMP-131 [Oct09] [Aug10] [Sep11] [Aug12] Duration of Troponin elevation post myocardial infarction (also asked as cTnI)

A. 12-24 hours
B. 24-48 hours
C. 2-5 days
D. 5-14 days
E. 2-4 weeks
D. 5-14 days
--
After myocardial cell damage, unbound cytoplasmic troponin is released from cardiac myocytes. Both troponin I and T exhibit biphasic release kinetics. Release from the cytosolic pool gives increase to blood concen- trations rising 4 – 6 h after the onset of damage and peaking at 12 – 24 h after myocardial injury. Structural protein release leads to a second peak 2 – 4 days after injury. Continuing breakdown of myofibrillary-bound complex explains the prolonged elevation of both troponins for up to 10 days after infarction.
-----------
TMP-Jul10-002

Male with a Haemoglobin of 80g/L and reticulocyte count 10%. Possible diagnosis:

A. Untreated pernicious anaemia

B. Aplastic anaemia

C. Acute leukaemia

D. Anaemia of chronic disease

E. Hereditary spherocytosis
E. Hereditary spherocytosis

All the rest you have a low reticulocyte count and in spherocytosis the reticulocyte count is between 6-20% average of 10%
-----------
TMP-Jul10-015 [Aug12]

Which type of aortic dissection is classically for NON-operative management:
A. DeBakey Type I
B. DeBakey Type II
C. Stanford A
D. Stanford B
E. Stanford C
D. Stanford B
--
Stanford A involves ascending aorta (DeBakey types I and II);
Stanford B involves the descending aorta below the subclavian artery (DeBakey III) and has traditionally been managed with medication.

Stanford C does not exist?
-----------
TMP-Jul10-020 [Aug12]

Thermoneutral zone in 1 month old infant ?
A. 26 – 28 degrees Celcius
B. 28 – 30 degrees Celcius
C. 30 – 32 degrees Celcius
D. 32 – 34 degrees Celcius
E. 34 - 46 degrees Celcius
D. 32 – 34 degrees Celcius
-
From wiki
The thermoneurtral zone is didined as the environmental temperature range over which metabolic rate is kept at a minimum and within which temperature regulation is achieved by non-evaporative physical processes alone. This zone is much higher in the naked neonate (32-36 deg C) than in name adults.
-----------
TMP-Jul10-043 [Mar11][Sep11][Aug11]

Young asthmatic male in emergency department. RR 26, pCO2 27, SAO2 92%, struggling talking in sentences. Given nebulised salbutamol, and ipratropium, 200mg IV hydrocortisone. After 30 minutes - no improvement. Further management:

A. IV salbutamol
B. IV aminophylline
C. IV magnesium
D. Intubate and ventilate
E. ???IV adrenaline?
?helium/oxygen mixture also option in other stem?
C. IV magnesium

See Global Initiative for Asthma 2012 update figure 4.4-2 Management of Asthma exacerbations in acute care setting. Intravenous magnesium is part of treatment for severe episode (along with oxygen, inhaled b2 agonist and inhaled anticholinergic and systemic glucocorticosteroids.)

-----------
TMP-Jul10-046 [Aug12]

Long-standing T6 paraplegia. All present EXCEPT ?
A. Flaccid paralysis
B. Poikilothermia
C. Autonomic hypereflexia
D. Labile BP
E. Hyperkalaemia with suxamethonium
A. Flaccid paralysis
--
a. flaccid paralysis - false - get spasticity so this is the answer to choose.
b. poikilothermia - true - unable to regulate own temperature
c-e. all true.
-----------
TMP-Jul10-048
[Mar11][Sep11][?Aug12]
Amniotic fluid embolism. Cause of death in first half hour ?
A. Pulmonary hypertension
B. Malignant arrhythmia
C. Pulmonary oedema
D. Hypovolaemic shock
E.
A. Pulmonary hypertension
-
From CEACCP article on AFE 7(5) p 152
' Amniotic fluid and fetal cells cause an increase in both systemic and pulmonary vascular resistances resulting in acute pulmonary hypertension. Survivors of this response develop left ventricular failure and pulmonary oedema '
-----------
TMP-Jul10-052
[Sep11][?Aug12]
Nerve block for anaesthesia over anterior 2/3 of ear?
A. C2
B. Mandibular nerve
C. Maxillary nerve
D. Ophthalmic nerve
E. Vagus
B. Mandibular nerve [?Auriculotemporal nerve in Aug12 stem]

Mandibular n. is largest division of the trigeminal n. with sensory roots from the trigeminal ganglium and motor roots from the pons and medulla.

Sensory nerves from posterior division include
- AURICULOTEMPORAL n. which is sensory to EXTERNAL AUDITORY MEATUS and external surface of the tympanic membrane.
- Lingual n. - provides sensation to ant 2/3 of tongue, floor of the mouth and lingual gingiva
-----------
TMP-Jul10-056
[Mar11][Aug12]

Thallium scan:
A. High negative predictive value
B. High positive predictive value
C. Not as good as a dobutamine stress echocardiography
D.
E.
A. High negative predictive value
-
Thallium scanning in coronary artery disease (CAD) using radioactive thallium-201(201-Tl) is a widely available technique which is sensitive, accurate and noninvasive. It detects CAD accurately in patients with:

atypical chest pain and a positive exercise ECG or
typical chest pain and a negative exercise ECG.

In known CAD, it determines the severity, extent and haemodynamic significance of a stenosis.

In patients with recurrent chest pain following coronary artery graft surgery (CAGS) or angioplasty, scanning can detect the presence and severity of any ischaemia, when graft occlusion or restenosis has occurred.

201-Tl has important prognostic value in predicting cardiac complications (death and acute myocardial infarction) in patients undergoing major surgery - usually vascular.

In patients with acute myocardial infarction, it may contribute to risk stratification.

-from Aust Prescr 1994;17:57-61

Possibly 'A' depending on the stem. In the recent vascular surgery webinar/podcast Dr Machlin emphasised that, in the context of using perfusion scans to predict perioperative ischaemic events, a negative study was of much more benefit i.e. they have a high NPV but a low PPV. No one perfusion scan was any better than the others, it just depended what was available in your institution.SGB

From the 2007 AHA guidelines,

'because of a very high sensitivity of abnormal stress nuclear imaging studies for detecting patients at risk for perioperative cardiac events, the negative predictive value of a normal scan has remained uniformly high at approximately 99% for MI or cardiac death'

-----------
TMP-Jul10-058 [?Aug12]

Young female having cholecystectomy. Venous air embolus:
A. Mechanical ventilation and PEEP is part of treatment strategy
B. Most likley to occur at initial gas insufflation, but can occur at any time
C. Inert gas (argon, xenon) is safer
D.
E.
B. Most likley to occur at initial gas insufflation, but can occur at any time
--
"This complication develops principally during the induction of pneumoperitoneum, particularly in patients with previous abdominal surgery." Miller 7th ed. p. 2188.
-----------
TMP-Mar10-079
[Aug10-123][?Aug12]
Head Trauma patient with unilateral dialated pupil, whats the diagnosis ?

A.Global injury
B.Optic nerve injury
C.Horners syndrome
D.Transtentorial herniation
E.
D. Transtentorial herniation
-----------
TMP-Mar11-027
[Mar12][Aug12]
Popliteal block placed from the lateral approach:
A: Passes through semimembranosus
B: Has eversion of the foot as the end point for nerve stimulation
C: Has increased failure rate compared to a posterior approach
D: Adequate for ankle surgery (Aug12)
E: Can be performed supine or prone
E: Can be performed supine or prone
--
A: false - semimembranous is medial (see nysora website on popliteal block)
B: false - common peroneal nerve stimulation results in dorsiflexion and eversion. stimulation of the tibial nerve results in plantar flexion and inversion;
From Reg Anesth Pain Med 2010;35: S16-S25: "dorsiflexion or eversion indicating common perineal nerve stimulation, plantar flexion indicating TN stimulation, and INVERSION indicating simultaneous stimulation of both branches). It has been demonstrated that inversion may be the optimal single evoked motor response to maximise the onset and success of sciatic nerve block"
C: false i think - it would depend if using nerve stimulator or US.
D: false - ?need to block saphenous n. as well especially if tourniquet is used (see frca.co.uk)
E: can be performed supine or prone - definitely true. (especially with US, prone with nerve stimulator so can see twitches better)
-----------
TMP-Mar11-028
[Mar12][Aug12]
During scoliosis surgery with monitoring of somatosensory evoked potentials, which tract are they mainly monitoring?
A: Dorsal column
B: Spinothalamic tract
C: Lateral Corticospinal tract
D: Cerebrospinal tract
E: Anterior horn cells
A: Dorsal column
--
From www.scoliosisjournal.com/content/5/1/8
"Somatosensory evoked potentials (SSEP) measure the integrity of the sensory pathways in the dorsal columns of the spinal cord, by stimulating a peripheral sensory nerve and measuring the electrical response in the brain. The introducetion of SSEP monitoring to spinal surgery has significantly reduced the rate of intraoperative injury."
-----------
TMP-Mar11-031
[Sep11][Mar12][?Aug12]
NEW. Which of the following causes the most heat loss in a neonate?

A. vasodilatation
B. radiation
C. convection
D. conduction
E. evaporative
B. radiation
-----------
TMP-Mar11-032
[Aug12]
75 year old with non-valvular AF usually on warfarin has their warfarin stopped for one week. What is their daily risk of stroke?
A: 1%
B: 0.1%
C: 0.01%
D: 4%
E: 10%
C 0.01%
--
CHADS2 (CCF, Hypertension, Age ≥75 years, DM, Stroke/TIA or Thromboembolism)
- pts score would be 1 which gives annual stroke risk of 2.8%
- does that translate to a daily stroke risk of ~0.01% ?
CHA2DS2-VASc (CCF, HT, Age ≥75 years, DM, Stroke/Tia or thromboembolism, Vascular disease (IHD, PVD etc), Age 65-74, Sex category (female))
- pts score would be 3 - scores 2 for being 75 - so his annual stroke risk would be 2.2%
-----------
TMP-Mar11-036
[Mar12][Aug12]
What is the major cause of death in a patient with perforation of the pharynx, oesophagus or trachea?
(Or: Major cause of death following difficult intubation with perforated oesophagus)

A: failure to intubate
B: failure to ventilation
C: sepsis
D: bleeding
C: Sepsis
(though previous grp said B)
--
From AANA Journal/June 2005/Vol 73 (3):
'The AANA and ASA closed claims analysis underscores seriousness of pharyngoesophageal perforation. Despite the relatively rare occurrence of this type of injury compared to other airway injuries, pharyngoesophageal perforation, especially when diagnosis and treatment were delayed, was associated with the poorest outcome. The late sequelae of undiagnosed pharyngoesophageal perforation are mediastinitis, retropharyngeal abscess, pneumonia, pericarditis, and death. Earlier signs are pneumothorax and pneumomediastinum (Table 1).'
-----------
TMP-Mar11-041
[Aug12]
75 year old male with normal renal function for an endoluminal aortic repair. What is the best protection to prevent the development of renal dysfunction?
A: NaCl
B: NAC
C: mannitol
D: dopamine
E: dialysis
A: NaCl - adequate fluid loading should be ensured by administration of intravenous crystalloid for about 12 hours prep and continuing after the procedure for 6 to 24 hours.
-
B: false - CEACCP Periop renal protection article 2008 - No evidence for N-acetyl cysteine of open AAA or endoluminal repair.

C: false - from J Endovasc There 2008;15:73-82 - 'Mannitol reduces renal damage during open AAA repair. HOWEVER it INCREASES the risk of complications if given with contrast media.'

D: false - from CEACCP article above - 'Dopamine infusion has not been shown to prevent ARF, avert the need for renal replacement therapy, or reduce mortality, and should not be administered solely for renal protection'.
-----------
TMP-Mar12-002
[?Aug12]
Which of the following is NOT a side effect of cyclosporine
a. Alopecia
b. Hypertension
c. Renal impairment
d. Gum hyperplasia
A. Alopecia
-
from www.rheumatology.org.au.
Most common side effects
- increase and darkening of body hair
- bleeding, tender or enlarged gums
- pins and needles in hands and feet. Slight tremor of hands though these tend to lessen after a few weeks
- renal impairment
- hypertension
-----------
TMP-Mar12-003 [Aug12]

What is the half life of clopidogrel?

a. 6 hours

b. 14 hours

c. 24 hours

d. 7 days
a. 6 hours

From Product Information of clopidogrel:
After a single oral dose of 75mg, clopidogrel has a half-life of approximately 6 hours. The elimination half-life of the main circulating (inactive) metabolite was 8 hours after a single and repeated administration. The half life of the active metabolite is about 30 minutes.


Clopidogrel is a prodrug. It is rapidly absorbed (at least 50%). Mean peak plasma levels of unchanged drug occur ~45mins after dosing. It is 98% protein bound.

Metabolism occurs in liver via two pathways - one by esterases leading to hydrolysis to an inactive metabolite (85% of circulating metabolites) and one mediated by multiple cytochrome p450.

Cytochromes first oxidise clopidogrel to a 2-oxo-clopidogrel intermediate metabolite. Subsequent metabolism results in formation of the active metabolite, a thiol derivative of clopidogrel. This metabolic pathway is mediated by CYP2C19, CYP3A, CYP2B6 and CYP1A2. The thiol metabolite binds rapidly and irreversibly to platelet receptors thus inhibiting platelet aggregation for the lifespan of the platelet.
-----------
TMP-Mar12-007 [Aug12]

Which of the following is not included in the CHADS2 AF thromboembolic risk scoring system

a. Age

b. Gender

c. Diabetes

d. Heart failure

e. Previous TIA
b. Gender
-
CHADS2 = Congestive heart failure, Hypertension, Age> or = 75, Diabetes, previous Stroke or TIA or Thromboembolism (double weight).

CHADS2 score is a clinical prediction rule for estimating risk of stroke in patients with non-rheumatic AF and is used to determine whether or not treatment is required with anticoagulation or antiplt tx.

now superseded by CHA2DS2-VASc
- Congestive heart failure (or LV systolic dysfunction)
- Hypertension
- Age > or = 75 (double weight)
- Diabetes
- Stroke or TIA or thromboembolism (double weight)
- Vascular disease (e.g. peripheral artery disease, MI, aortic plaque)
- Age 65-74
- Sc = sex category (female=1)
-----------
TMP-Mar12-014

Where should the tip of an IABP lie

a. 2cm distal to the left subclavian

b. 2 cm proximal to the left subclavian

c. 2cm proximal to the renal artery

d. 2 cm distal to the renal artery
a. 2cm distal to the left subclavian

-----------
TMP-Mar12-015
[Aug12]
A 60kg female is given 50 mg of rocuronium, she is unable to be intubated, what dose of sugamadex is required to reverse the rocuronium

a. 240

b. 800

c. 960
c. 960
--
Dosing for sugammadex
2mg/kg for moderate block
4mg/kg for profound block
16mg/kg for immediate reversal.

from BJA 2010 systematic review:
Moderate block - return of T2 or T1 20% or 25%
Profound block - PTC 1-2
-----------
TMP-Mar12-018 [Aug12]

What is a contraindication to an IABP?

A. Aortic regurgitation

B. Aortic stenosis
A. Aortic regurgitation
--
CEACCP 2009 article:
CONTRAINDICATIONS
Absolute - AR, aortic dissection, Chronic end-stage heart disease with no anticipation of recovery, aortic stents
Relative - uncontrolled sepsis, AAA, tachyarrhythmias, severe peripheral vascular disease, major arterial reconstruction surgery

INDICATIONS
AMI, cardiogenic shock, Acute MR and VSD, Catheterization and angioplasty, refractory unstable angina, refractory LV failure, refractory ventricular arrhythmias, cardiomyopathies, sepsis, infants and children with complex cardiac anomalies.
-----------
TMP-Mar12-022
[Aug12]

An epidural in a healthy individual causes all EXCEPT

a. Raised Co2

b. Bradycardia

c. Vasodilation

d. Dyspnea

e. reduced circulating catecholamines [in Aug12]
a. Raised Co2 - ?false and answer to choose
--
a. From Textbook of regional anaesthesia and acute pain management Ch 14 - epidural blockage - see Table 14-5. Significant REDUCTION in PaCO2 AFTER thoracic epidural.

b. and c. - From above reference:
Block below T4 - vasomotor tone is maintained by sympathetic fivers from T5-L1 that innervate vascular smooth muscle. Blockade of these fibers causes venodilation and arterial vasodilation with decreased SVR. This leads to reduced venous return, RAP and subsequently reduced CO. However upper body vasoconstriction with baroreceptor activation can lead to increased vagal tone, causing bradycardia DESPITE the decrease in venous return. (Bezold-Jarisch)
Block above T4: high sympathetic block, cardiac symp fibers arise from T1-T4 so when blocked, profound hypotension and bradycardia can occur.
-----------
TMP-Mar12-024
[Aug12]

Autologous transfusion results in less

a. Cost

b. Blood waste

c. Incompatible transfusion

d. Unrequired transfusion
d. unrequired transfusion?? - true because patient is more likely to require blood transfusion especially if preop autologous donation or acute normovolaemic hemodilution;
--
a. false - autologous transfusion may require capital expenditure, increased use of disposables and staff training
b. false - unused blood is wasted and cannot be returned to the donor pool. from transfusion.com.au 50% of autologous units are discarded (with PAD not ANH)
c. ?false - complex logistics for collection, storage and transfusion of the correct unit to the appropriate patient may increase potential for clerical error
d. ?true - see above

see Miller Ch 57 or 2006 CEACCP article.
-----------
TMP-Mar12-026 [?Aug12]

Bleeding in trauma has been shown to be reduced by

a. Tranexamic acid

b. Recombinant factor VIIa

c. DDAVP

d. Prothrombinex
a. Tranexamic acid
--
a. Massive Transfusion Consensus Conference 2011 (http://ccforum.com/content/15/6/242) unanimous agreement:
1. Early (up-front) administration of tranexamic acid (1g over 10 minutes and then 1 gram over 8 hours within 3 hours of injury
2. For patients with critical bleeding, immediate application of a 'foundation ratio' of blood components. Eg 6 RBCs and 3 FFP for initial treatment
3. Adjustments to the foundation ratio of transfusion support based on clinical course and lab results using goal-directed blood therapy.
CRASH-2 trial have shown significant reduction in death due to bleeding and all cause mortality in trauma patients receiving tranexamic acid (if given before 3 hours). Maybe harmful if given after 3 hours.

b. rVIIa - false; from same reference "There is evidence from more than one randomized controlled trial to support the lack of utility with the use of recombinant activated factor VII (rVIIa) for the treat- ment of bleeding in blunt trauma or penetrating trauma patients [16-18]. rVIIa is not licensed for nor recommen- ded for the prevention or management of hemorrhage in trauma patients."

c. false - from http://ccforum.com/content/14/2/R52 European guidelines 2010 managing bleeding in trauma - recommendation 30: We do not suggest that desmo- pressin be used routinely in the bleeding trauma patient (Grade 2C). We suggest that desmopressin be consid- ered in refractory microvascular bleeding if the patient has been treated with platelet-inhibiting drugs such as acetylsalicylsalicylic acid (Grade 2C).

d. false - from european guidelines above: We recommend the use of pro- thrombin complex concentrate for the emergency reversal of vitamin K-dependent oral anticoagulants (Grade 1B). There is a current lack of data with its use outside of this recommendation.
-----------
TMP-Mar12-027 [Aug12]

The time constant of the lung is calculated by

a. Compliance x resistance

b. Compliance plus resistance

c. Compliance /resistance

d. Resistance/compliance
a. compliance x resistance

Compliance - measurement of lung distensibility and expressed as change in volume divided by change in pressure. Normal lung+thorax compliance is 0.1L/cmH2O. Low compliance - more pressure will be needed to deliver given volume (e.g. ARDS). Emphysema is typical of increased lung compliance

Resistance - pressure needed to deliver flow of gas and expressed as change in pressure divided by flow. Normal is 0.5-1.5 cmH2O/L/sec.

Time Constant of the lung is a phenomenon whereby a given percentage of passively exhaled breath will require a constant amount of time to be exhaled regardless of the starting volume given constant lung mechanics. Mathematically is compliance multiplied by airway resistance and resulting value has units of seconds.

http://www.lexingtonpulmonary.com/education/lungmech/lungmech.html
-----------
TMP-Mar12-032 [Aug12]

What is the ratio of MAC awake:MAC of sevoflurance

a. 0.2
b. 0.34
c. 0.5
b. 0.34
--
ref: 'Cerebral Awakening Concentration of Sevoflurane and Isoflurane Predicted during Slow and Fast Alveolar washout' Anesthesia and Analgesia 1993.
Mac awake for sevo was the same for both slow and fast washout.
-----------
TMP-Mar12-035 [Aug12]

The features of Pierre Robin sequence include cleft palate, micrognathia and:

A. Glossoptosis
B. Craniosynostosis
C. Macroglossia
D. Microstomia
A. Glossoptopsis

From Stoelting Anaesthesia and Coexisting disease
"Pierre Robin syndrome consists of micrognathia usually accompanied by glossoptosis (posterior displacement of the tongue) and cleft palate. Mandibular hypoplasia may be responsible for displacement of the tongue into the pharynx, which subsequently prevents fusion of the palate. Acute upper airway obstruction can occur in neonates or infants with Pierre Robin syndrome. Feeding problems, failure to thrive, and cyanotic episodes are other early complications of this syndrome. Associated congenital heart disease is frequent. Fortunately, sufficient mandibular growth during early childhood markedly reduces the degree of airway problems in later years."
-----------
TMP-Mar12-038a [Aug12]

After a procedure with an LMA in situ a patient complains of loss of sensation to the anterior part of the tongue. What nerve is likely damaged?

a. Facial
b. Lingual
c. Greater palatine
d. Glossopharyngeal
b. Lingual > a. Facial
--
Just before entering this foramen, the FACIAL nerve gives off the chorda tympani, which pierces the posterior wall of the tympanic cavity close to the deep surface of the ear drum. It runs forward over the pars flaccida of the tympanic membrane and the neck of the malleus, lying immediately beneath the mucous membrane throughout its course. It passes out through the front of the middle ear by piercing the bone at a canaliculus at the inner end of the petrotympanic fissure. It emerges from this fissure to join the LINGUAL nerve about 2.5 cm below the base of the skull. Through the chorda tympani, taste fibres are conveyed from the anterior two-thirds of the tongue and secretomotor fibres reach the submandibular ganglion.

The literature only speaks of damage to LINGUAL nerve (as strictly speaking this is the nerve that is damaged though it carries fibers from the FACIAL nerve). Other nerves that may be damaged is hypoglossal or recurrent laryngeal.

see: BJA 2005 'Lingual nerve injury associated with the ProSeal laryngeal mask airway: a case report and review of the literature'
-----------
TMP-Mar12-038b [Aug12]

Patient complains of numbness of the anterior third of his tongue following GA with LMA. Which nerve is involved?

A. Glossopharyngeal
B. Facial nerve
C. Superior vagus
D Mandibular n.
B. Facial nerve
--
Just before entering this foramen, the FACIAL nerve gives off the chorda tympani, which pierces the posterior wall of the tympanic cavity close to the deep surface of the ear drum. It runs forward over the pars flaccida of the tympanic membrane and the neck of the malleus, lying immediately beneath the mucous membrane throughout its course. It passes out through the front of the middle ear by piercing the bone at a canaliculus at the inner end of the petrotympanic fissure. It emerges from this fissure to join the LINGUAL nerve about 2.5 cm below the base of the skull. Through the chorda tympani, taste fibres are conveyed from the anterior two-thirds of the tongue and secretomotor fibres reach the submandibular ganglion.

The literature only speaks of damage to LINGUAL nerve (as strictly speaking this is the nerve that is damaged though it carries fibers from the FACIAL nerve). Other nerves that may be damaged is hypoglossal or recurrent laryngeal. The facial nerve is 'INVOLVED' - depends how question was actually worded!

-----------
TMP-Mar12-040 [aug12]
see also EZ94

A man is working with electrical appliances at home with a residual current device. If he touches the active and the neutral (was it neutral or earth) wire he will suffer

a. A microshock
b. A macroshock
c. Nothing happens because the fuse blows
d. The RCD will protect him from macroshock
b. A macroshock

?d. The RCD will protect him from macroshock (if he doesn't touch the wires at the same time… see below)
--
From 'Electrical hazards causes and prevention' Anaes Int Care Med:
A residual current device is much more sensitive and can break a circuit before a person perceives any pain. This relies on the principle that normally in a correctly working system there is a balance of current in the live and neutral leads. In a fault situation, there is an imbalance of current and this causes an electrically operated switch to break the circuit.

HOWEVER - if he touches active an neutral wire AT THE SAME TIME - no difference in current is detected and the current will pass through him causing a shock.

From http://www.allstatesafety.com.au/services/rcd-residual-current-device-safety-switch/
What an RCD will not protect;
If a user was to touch both active and neutral at the same time, then current flow would be even through both circuits. RCD WON’T TRIP
Scenario #1 A carpenter using a tool with an internal fault, can be electrocuted from a positive to neutral connection, because both positive and neutral would be feeding electricity evenly into the body, hence the RCD wouldn’t trip.
Scenario #2 An ice machine with a faulty earth and a positive short to it metal frame is touched by a user; positive could flow into the users hands join the neutral side within a fan or compressor. This would happen because an earth wasn’t present. Electrons always take the path of least resistance. If a good earth contact was present then as soon as a positive short was made to the frame positive would have gone to earth and its amperage would have been higher than the negative, therefore the RCD would have tripped.
In basic terms if you short;

• Positive to earth – RCD Trips
• Positive to negative with a good earth – RCD Trips
• Positive to negative – RCD wont trip
• Positive to a metal frame with no earth – RCD wont trip
-----------
TMP-Mar12-045 [Aug12]

A patient in recovery post op total hip replacement develops crushing central chest pain, ECG shows ST segment elevation (NB- no BP etc given, beta blockade was not an option). The most appropriate action is to give

a. Aspirin
b. IV GTN
c. IV heparin
d. Calcium channel blocker
e. T/L
a. Aspirin
or ??d. Calcium channel blocker
--
a. With regards to aspirin, STRATAGEM trial found no difference with regards to thrombotic events or bleeding in patients who continued aspirin preoperatively for non cardiac surgery compared to patients who CEASED preop (though study is underpowered). Not sure how one can extrapolate these findings. PEP trial found trend toward a HIGHER risk of FATAL and non fatal MI in patients given periop aspirin…

d. with regards to Ca channel blocker. From Perioperative Myocardial Infarction published in Circulation 2009 - "None of the randomized trials has shown reduced PMI or death, despite less supraventricular arrhythmia and ischemia in thoracic surgery. A meta-analysis of 11 trials found reduced combined perioperative death or PMI by posthoc analysis but more hypotension with calcium channel blockers."

-----------
TMP-Oct09-035 [Aug12]

(NEW) Emergence delirium in a kid in recovery. To treat
a. Fentanyl 1mc/kg
b. Midazolam 0.1 mg/kg
c. Propofol
d. Clonidine 1mc/kg
e. Sucrose
e. Sucrose
-
See wiki debate. There are studies regarding PREVENTION but not so much on TREATMENT.

EA is self limited and usually resolves without pharmacological intervention.
studies- fentanyl, propofol, midazolam but best is to reunite with parent.
-----------
TMP-Sep11-004 [Aug 12]

Child with murmur- what would make it more likely for you to investigate if you heard the murmur
A. persist in supine position
B. louder or softer with various manouveres

[aug12]
A. 4/6 loudness
B. ????vibratory/flutter sound
A. 4/6 loudness (Aug12 version)
--
From http://www.aafp.org/afp/1999/0801/p558.html
"Characteristics of pathologic murmurs include a sound level of grade 3 or louder, a diastolic murmur or an increase in intensity when the patient is standing".

Also from Am J Cardiol. 2008 Oct 15;102(8):1107-10. Samuel A. Levine and the history of grading systolic murmurs:
"Of 19 subjects with grade 3 or 4 murmurs, all were determined to have organic heart disease or anemia. Thus, louder systolic murmurs were found to be a significant finding, as were the cause location and effects of posture".

1/6: very soft and not heard at first
2/6: soft, but can be detected almost immediately by an experienced auscultator
3/6: moderate; no thrill
4/6: loud; thrill just palpable
5/6: very loud; thrill easily palpable
6/6: very very loud; can be heard without placing stethoscope on the chest!

See Table 1 CEACCP article:

Innocent - Asymptomatic, early systolic or continuous (venous hum), blowing/musical/vibratory in quality, No precordial thrill, often varies with posture

Pathological -
symptomatic (hx recurrent chest infections, cyanosis, tachypnoea, sweating, feeding difficulties, failure to thrive),
Diastolic, pan systolic or late systolic
Variable/harsh in quality
precordial thrill sometimes present
Rarely varies with posture (HOCM murmur increases with standing)

ALSO -
Most CHD is identified before 3 months old but any child under 1 yr with a murmur should be referred to a paediatric cardiologist before anaesthesia, even if asymptomatic as significant lesions may be slow to present.
Be suspicious of children with syndromes assoc with CHD
- Down's syndrome
- CHARGE (coloboma of eye, heart defects, atresia of choanae, retardation, genital/urinary abnormalities, ear abnormalities/deafness)
- VATER (vertebral anomalies, anal atresia, TOF, radial dysplasia)
- Turner's
- DiGeorge
In older children watch for decreased extol, squatting during play (tet spells), syncope -- serious heart disease
Family history of sudden death -- suspect HOCM, has autosomal dominant inheritance but often asymptomatic.
ECG - look for ventricular hypertrophy

In an asymptomatic child over 1 yr old with an innocent murmur and normal ECG -- probably safe to proceed with surgery and refer for investigation after operation
-----------
TMP-Sep11-009b
[Mar12][Aug12]
Which is the best predictor of poor prognosis with aortic stenosis?
A. chest pain
B. paroxysmal nocturnal dyspnoea
C. syncope
D. palpitations
E. fatigue
B. paroxysmal nocturnal dyspnoea

Patients die within an average of 5 yrs after onset of angina, 3 yrs after onset of syncope and 2 years after onset of heart failure symptoms.
-----------
TMP-Sep11-029
Subarachnoid haemorrhage patient. What percentage rebleed in the first 24hours
A. <5%
B. 5-10%
C. 10-15%
D. 15-20%
E. >20%
A -- according to Oxford Handbook of Anaesthesia

HOWEVER:
According to BJA 2007 article (Aneurysmal SAH and the anaesthetist) Table 5 - rebleeding risk on day 1: 15%
According to CEACCP 2012 article (Acute management of aneurysmal SAH) - risk of rebleeding is greatest immediately after the initial haemorrhage with rates of 5-10% within the first 72h.
-----------
TMP-Sep11-047 [Aug12]
see TMP-Jul10-043 ABG results are different!

Severe asthmatic- tachycapnia, HR120, speaking in words, pH 7.45, pCO2 46, pO2 96, HCO3 24. Then given nebulised salbutamol continuously, nebulised ipratropium bromide, and hydrocortisone- The next step:

A. ?
B. ?
Describes Acute Severe Asthma (previously known as status asthmaticus). of concern is rising CO2 -- ?fatigue

needs IV magnesium. based on CO2>45
requires ICU admission
continue inhaled b2 agonists and anticholinergics and IV glucocorticosteroids
consider IV b2 agonist
consider IV theophylline
possible intubation and mechanical ventilation (see GINA 2012 figure 4.4-2)
-----------
TMP-Sep11-051
[Aug12]

Post cervical spine op, there is bulging noted under the incision sit:E. Patient desaturated, combative, keep pulling off the oxygen facemask. Next course of action
A. Rapid sequence induction
B. Gas induction
C. Needle aspiration of the bulge at the neck
B - with surgeons ready to cut the throat.

Ref: Airway compromise due to laryngopharyngeal edema after anterior cervical spine surgery. Journal of Clinical Anesthesia. epub ahead of print June 2012

?depends on how soon post op. Immediate postop (<12hrs) maybe due to haematoma. Early (12-72hrs) due to pharyngeal/prevertebral edema. Late due to abscess, csf accumulation, construct failure. Would C be of benefit if due to haematoma???
-----------
TMP-Sep11-072
[Mar12][Aug12]

Modified Cormack and Lehane grade - You cannot see beyond the epiglottis and there is a little space between the epiglottis and the posterior pharyngeal wall (? remembered as epiglottis touching posterior pharyngeal wall)
A. 2a
B. 2b
C. 3a
D. 3b
E. 4
C or ?D (prev grp answer)

--
From Evidence Based Practice Anaesthesiology Ch17:
- 2a - partial view of glottis visible
- 2b - only arytenoids visible (2b is associated with greater difficulty)
- 3a - epiglottis can be seen and lifted
- 3b - epiglottis seen but CANNOT be lifted

6 grades and 3 functional classes:
easy: laryngeal inlet visible and indubitable under direct vision (1 and 2a)
restricted: posterior glottic structures visible or epiglottis visible and can be lifted (2b and 3a) these views likely to benefit from indirect intubation methods
difficult: epiglottis cannot be lifted or when no laryngeal structures are visible - likely need specialist methods for intubation and may need to be performed blindly (3b and 4)
-----------
TMP-Sep11-082 [Mar12][Aug12]

Atrial Septal Defect (ASD) murmur is due to flow through which valve?
A. ASD
B. Tricuspid valve
C. Pulmonary valve
D. Mitral valve
E. Aortic valve
C. Pulmonary valve

(where is the ASD spot?)
--
From Talley O'Connor 5th ed.
ASD:
- Ostia secundum (90%) where defect does not involve AV valves and ostia primum where defect involves AV valves
- Auscultation: fixed splitting of S2, defect produces no direct murmur but increased flow through R side of heart can produce low pitched diastolic tricuspid flow murmur and more often a pulmonary systolic ejection murmur louder on inspiration.
-
From emedicine:
- Findings on examination depend on degree of L-R shunt and hemodynamic consequences.
- Hyperdynamic RV impulse due to increased diastolic filling and stroke volume.
- Palpable pulsation of PA and ejection click can be detected because of a dilated pulmonary artery
- S1 is typically split and second component may be increased in intensity due to forceful RV contraction and delayed closure of TV
- S2 is often widely split and fixed due to reduced respiratory variation from delayed PV closure (only seen if PAP is normal and PVR is low). This is the CHARACTERISTIC abnormality found in almost ALL pts with large L-R shunts.
- blood flow across ASD does not cause a murmur at site of shunt because no substantial pressure gradient exists between the atria.
- however ASD with moderate-large L-R shunts -> pulmonary systolic ejection murmur (crescendo-decrescendo) due to increased stroke volume rather than any valvular problems.
- large L-R shunts often have rumbling mid diastolic murmur at TV because of increased flow.

Auscultatory findings may resemble those of mild valvular or infundibular pulmonic stenosis and idiopathic dilation of pulmonary artery.
- these all produce a systolic ejection murmur but DIFFER from ASD by movement of S2 with respiration, a pulmonary ejection click or absence of a tricuspid flow murmur.
-----------
TMP-Sep11-087
[Aug12]

Apnoeic oxygenation in obese patient can be increased by
A. Sniffing position
B. Prone
C. Supine
D. Lateral
E. Head up
E. Head up
-
See 2005 BJA article which compared sitting (head up) to supine.
-----------
TMP-Sep11-090

Post partum sudden collapse, suspected amniotic fluid embolism. The consistent finding is:
A. Low C3, C4
B. Increase complement
C. Increase tryptase
D. Increase histamine?
E. petechial rash

[Aug 12 variation - finding against AFE]
A. Low C3,C4
--
See 2007 CEACCP article and also AFE article from Clinical and Developmental Immunology 2012.

Sialyl Tn antigen (Sialyl is a mucin-type glycoprotein that originates in feral and adult intestinal and respiratory tracts - major component in meconium and present in clear amniotic fluid) - is raised

Zinc Corprophyrin (component of meconium) - raised.
-----------
TMP-Sep11-105
[Mar12][?Aug12]

The cause of hypoxia in one lung ventilation
A. Blood flow through non ventilated lung
B. Impairment of hypoxic pulmonary vasoconstriction
C. Ventilation perfusion mismatched (?)
A. Blood flow through non ventilated lung

"The major cause of hypoxemia is the shunt of de-oxygenated blood through the non-ventilated lung. " - from www.thoracic-anesthesia.com by Peter Slinger
-----------
TMP-Sep11-121 [aug12]
New
Periop clinic reviewing a patient with chronic/ end stage renal failure. Her calcium found to be low. He most certainly have
A. Primary hyperparathyroidism
B. Secondary hyperparathyroidism
C. Tertiary hyperparathyroidism
B. Secondary hyperparathyroidism
--
Primary hyperPTH - unregulated overproduction of PTH resulting in abnormal homeostasis --> hyperCa

Secondary hyperparathyroidism - overproduction of PTH secondary to chronic abnormal stimulus for its production typically due to CRF. Another common cause is Vit D deficiency. In CRF - hypocalcaemima, hyperphosphataemia, impaired Vit D production.

Tertiary hyperPTH - state of excessive PTH secretion after longstanding secondary hyperPTH --> hyperCa. Can occur post renal transplant.
-----------
TMP-Sep11-131
[Mar12][Aug12]
New
When stimulating the ulnar nerve with a nerve stimulator, which muscle do you see twitch?

A. opponens abducens

B. abductor pollicis brevis

C. adductor pollicis brevis

D. extensor pollicis

E. flexor pollicis brevis
C. adductor pollicis brevis
-----------
TMP-Sep11-132
[Mar12][Aug12]

When intubating over a bougie / awake fibreoptic, which direction do you rotate the tube to stop it catching on structures in the glottis

A. no change from normal

B. 90 degrees clockwise

C. 90 degrees counterclockwise

D. 180 degrees

E. try either direction
C. 90 degrees counterclockwise

Counter clockwise rotation turns bevel inferiorly. Tip passes under epiglottis, gap is closed and tube enters larynx symmetrically.
-----------
TMP-Sep11-138
[Mar12][Aug12]

A 62 year old man has chronic renal failure. You notice his total serum calcium is 2.05 mmol/L. This is because he has

A. high serum vitamin D

B. hypoparathyroidism

C. primary hyperparathyroidism

D. secondary hyperparathyroidism

E. tertiary hyperparathyroidism
?D or E?
--
Mar12 wiki exam someone voted D.

there is not enough clinical info in this remembered question. Most CRF patients have secondary hyperPTH (hypoCa) which may develop into tertiary hyperPTH (hyperCa due to prolonged parathyroid gland stimulation.
-----------
SG47 ANZCA version [2003-Aug] Q149, [2004-Aug] Q90, [2005-Apr][Aug12]

A 20 kilogram child suffered 15% full thickness burns 6 hours ago. Optimum crystalloid resuscitation for the first hour is

A. 160ml

B. 260ml

C. 360ml

D. 460ml

E. 660ml
C. 360ml


• Parkland formula (Fluid in first 24 hours) = % burn x kgs x 4

∴ Fluid required in first 24 hours is 15 x 20 x 4 = 1200mls.

Plan to give half of this (600mls) in first 8 hours after the burn then the 2nd half (600mls) in the next 16 hours.

As 6 hours has already passed, then the 600mls needs to be given in the 2 hours remaining in the first 8 hour period (This is what the detail about the Parkland formula says to do to catch-up).

So give 300 mls/hr in "the next hour" (& same in the hour after that)

• "Maintenance fluids" should also be added to this value.

Maintenance fluids for 20kg child = (4 x 10) + (2 x 10) = 60 mls/hr

Fluids in 1st hour = replacement fluids + maintenance fluids

CORRECT ANSWER: Fluids in first hour = 300 + 60 = 360 mls/hr.

Then give the remaining 600mls over the next 16 hours - ie 37.5 mls/hour for 16 hours. BUT remember this is just an estimate and actual fluids given have to be adjusted based on urine output (aim for 1-1.5 mls/kg/hr in a child this age). (Est age for 20kg child is 6 yrs) and overall clinical assessment.
-----------
My comments:
from 'Burns in Children' CEACCP 2007 article, Table 1:
Parkland formula
For the first 24 h after the burn, give 4ml/kg per % BSA burn Hartmann's solution, half of this volume in the first 8 hour post-burn, the other half in the next 16 hours.
Maintenance - 4-2-1
Urine output - monitor and aim for 1mL/kg/hr

For this patient:
First 24 hours = 4x15x20 = 1200 mLs
Give 600 mLs in first 8 hours
PLUS maintenance of 60ml/hr - so in first 8 hours will need 480mLs of maintenance

So if this patient has had NO fluid resus - needs 600+(60x8) = 1080mLs in first 8 hours.

Can divide this volume over two hours - 540ml/hr??
PN42 [Mar 06] [Aug12]

When instructing ward staff on monitoring for respiratory depression in a patient using PCA (patient controlled analgesia) you would advise that early respiratory depression is best detected by monitoring

A. frequency of boluses on PCA machine

B. pulse oximetry

C. pupil size

D. respiratory rate

E. sedation scores
E. sedation scores
-----------